crim pro outline 1.1

90
Criminal Procedure—Bluth (Fall 2011) 1. ON STUDYING THE LEGAL REGULATION OF THE CRIMINAL JUSTICE PROCESS A. INTRODUCTION i. Defense atty’s job is to make the prosecution prove guilt beyond a reasonable doubt. ii. Prosecutor’s job is to ensure justice is done. iii. For the most part, criminal law is state law 1. State laws must NOT deprive rights that have been guaranteed by the US Constitution. 2. The law must also fall within the specific state constitution. B. THE STEPS IN THE PROCESS i. Pre-Arrest Investigation: How do we know if a crime has been committed? 1. Eye-witnesses, Victim, or Police Officer ii. Arrest: Seizure of a person 1. Officer may only detain a person if there is probable cause to do so (more likely than not that the crime was committed) 2. Miranda warnings given iii. Booking: Processing (booking) of the possible criminal 1. Take name, address, SS#, DOB, height, weight, hair color, fingerprints, DNA iv. Post-Arrest Investigation 1. Cops now talk to D, D doesn’t have to talk 2. Cops can bring in V and ask if this is the D v. The charging decision 1. This is usually when the prosecuting attorneys come in vi. Complaint filed 1. Prior to 1 st appearance, arrestee b/c an official “Defendant” vii. Magistrate Review of the Arrest 1. “Gerstein review” – magistrate determines that there exists probable cause for the offense charged in compliant viii. The First Appearance 1. Judge will tell the criminal why he has been arrested 2. Lawyer will be assigned if the criminal does not yet have one 3. Bond will be issued a. 8A guarantees the right to a reasonable bail in everything but capital cases i. This is designed to secure the appearance of the possible criminal ix. Preliminary hearing 1. Court sets preliminary hearing – a probable cause hearing in muni court, conducted by judge a. States don’t like doing this b/c they have to put witnesses on 2. or…Grand jury indictment a. 23 people decide whether there is probable cause, prosecution presents evidence and they are secret. GJ almost always indicts the D! x. Filing of the Indictment or the Bill of Information 1. Indictment or Information filed w/ trial court, takes place of complaint 1

Upload: keith-dyer

Post on 29-Oct-2014

109 views

Category:

Documents


3 download

TRANSCRIPT

Page 1: Crim Pro Outline 1.1

Criminal Procedure—Bluth (Fall 2011)

1. ON STUDYING THE LEGAL REGULATION OF THE CRIMINAL JUSTICE PROCESSA. INTRODUCTION

i. Defense atty’s job is to make the prosecution prove guilt beyond a reasonable doubt. ii. Prosecutor’s job is to ensure justice is done.

iii. For the most part, criminal law is state law1. State laws must NOT deprive rights that have been guaranteed by the US Constitution.2. The law must also fall within the specific state constitution.

B. THE STEPS IN THE PROCESS

i. Pre-Arrest Investigation: How do we know if a crime has been committed?1. Eye-witnesses, Victim, or Police Officer

ii. Arrest: Seizure of a person1. Officer may only detain a person if there is probable cause to do so (more likely than not that the

crime was committed)2. Miranda warnings given

iii. Booking: Processing (booking) of the possible criminal1. Take name, address, SS#, DOB, height, weight, hair color, fingerprints, DNA

iv. Post-Arrest Investigation 1. Cops now talk to D, D doesn’t have to talk 2. Cops can bring in V and ask if this is the D

v. The charging decision1. This is usually when the prosecuting attorneys come in

vi. Complaint filed1. Prior to 1st appearance, arrestee b/c an official “Defendant”

vii. Magistrate Review of the Arrest 1. “Gerstein review” – magistrate determines that there exists probable cause for the offense charged

in compliant viii. The First Appearance

1. Judge will tell the criminal why he has been arrested2. Lawyer will be assigned if the criminal does not yet have one3. Bond will be issued

a. 8A guarantees the right to a reasonable bail in everything but capital casesi. This is designed to secure the appearance of the possible criminal

ix. Preliminary hearing1. Court sets preliminary hearing – a probable cause hearing in muni court, conducted by judge

a. States don’t like doing this b/c they have to put witnesses on2. or…Grand jury indictment

a. 23 people decide whether there is probable cause, prosecution presents evidence and they are secret. GJ almost always indicts the D!

x. Filing of the Indictment or the Bill of Information 1. Indictment or Information filed w/ trial court, takes place of complaint2. In Ohio, the Information is used to plead to a lower charge

xi. Arraignment 1. D brought b/f trial court, informed of charges and asked to enter please of guilty, not guilty, or

nolo contendre2. Case transferred to judge who will preside over the case

xii. Pretrial motions (20 days to file)1. Improper venue, etc.2. If a motion is not filed, then it is waived

xiii. Guilty Plea Negotiation and Acceptance 1. 95% of all criminal cases will resolve w/o a trial, if plea process doesn’t work then trial

xiv. Then the jury will be selected1. Most places have 12 jurors2. The Constitution does not require a unanimous jury3. There are certain limits on the selection of a jury

xv. Case is finished and the criminal either walks or is then sentenced1. Sentencing has become extremely complex. Now have uniform sentencing, same facts, same

sentence. But these are just guidelines now, and judges can do what they want.xvi. Appeals (State pays for 1)

1

Page 2: Crim Pro Outline 1.1

1. Most criminal D’s appeal, they don’t pay for it, and there is no down-side for the Dxvii. Collateral Remedies

1. Post conviction procedures to challenge conviction (on the state side)2. Federal writ of Habeas Corpus, allows state prisoners to raise federal constitutional challenges to

their state convictions

2. SOURCES OF CRIMINAL PROCEDURE LAWA. CONSTITUTIONAL RIGHTS BINDING ON THE STATES:

i. The following rights have been held binding on the states under the due process provisions of the 14th Amendment:

1. 4th Amendmenta. Prohibits unreasonable searches and seizures b. The exclusionary rule requires that evidence obtained in violation of this prohibition

can’t be used against the D 2. 5th Amendment

a. D has a privilege against self-incrimination b. Prohibits double jeopardy

3. 6th Amendment a. D has a right to a speedy trialb. D has a right to a public trialc. D has a right to trial by jury for criminal chargesd. D has a right to confront witnessese. D has a right to compulsory process for obtaining witnesses f. D has a right to assistance of counsel in felony cases and in misdemeanor cases where

imprisonment is imposed4. 8th Amendment

a. Prohibits cruel and unusual punishment ii. Two provisions of the Bill of Rights not binding on the states:

1. The right to indictment by a grand jury for capital crimes. 2. Prohibition against excessive bail - Amendment 8.

____________________________________________________________________________________________________________

B. THE PROBLEM OF BODILY EXTRACTIONS

i. Rochin v. California (1952); Stomach Pump - Guy swallows two pills and the police have his stomach pumped to recover the evidence without his consent. S.C. held that pumping a D’s stomach by force and without his consent violates 14A due process.

1. Concerned about using such hazy standards, the concurring judges reasoned that this conduct also violated the 5A’s protection against self-incrimination.

II. WHEN IS IT OK FOR THE STATE TO PULL BLOOD OUT OF D’S VEIN? 1. If you know you would fail DUI test say NO – there is no upside to taking the test. 2. W/o blood test case much more difficult to prosecute….

a. W/o test prosecution must prove that alcohol affected your driving to a depreciable degree, he needs to prove bad driving – this is hard to do – not guilty.

3. But…NOW cops get judge to authorize a warrant to extract blood based on probable cause (more likely than not). D can later challenge the probable cause that the warrant based on.

iii. Class Hypo: Guy robs bar and gets shot in ass. Cops find D w/ bullet in ass and go to judge and ask for a warrant to have bullet removed. D doesn’t want surgery b/c it would prove guilt.

1. S.C. held judge does not have authority to gather evidence by ordering surgery. 4A, 5A, & 6A are limits on gov’t. They deliberately impede the gov’t from prosecuting criminals. The framers put these in place on purpose.

iv. D.A. v. Osborne ; (2009) Alaska, condom used in rape and sexual assault. D waived right to have condom tested at trial and D goes to jail. D then sued to force state to test the DNA on condom saying he had a constitutional right to demonstrate factual innocence, and state had no interest in denying this right. CoA agreed w/ D.

1. S.C. Held: Court of Appeals erred when it found that the inmate had a right under the DPC of 14th

A to have access to the evidence he sought. Only NEW evidence would qualify for DNA testing.

C. THE FEDERAL COURTS’ SUPERVISORY POWERS OVER THE ADMINISTRATION OF FEDERAL CRIMINAL JUSTICE

i. McNabb v. United States (1943); S.C. held incriminating statements obtained during prolonged and hence unlawful detention inadmissible in federal courts.

2

Page 3: Crim Pro Outline 1.1

1. While the S.C and other federal courts may reverse state convictions only on constitutional grounds, it may set procedural rules for federal prosecutions through its supervisory power over federal courts, “quite apart from the Constitution”

2. Purpose of supervisory powers: In the exercise of supervisory powers, federal courts may, within limits, formulate procedural rules not specifically required by the Constitution or the Congress. The purposes underlying use of the supervisory powers are threefold:

a. To implement a remedy for violation of recognized rights;b. To preserve judicial integrity by ensuring that a conviction rests on appropriate

considerations validly before the jury; andc. As a remedy designed to deter illegal conduct.

ii. STANDING LIMITATIONS (IRS used standing as a sword)1. Payner ; Gov’t stole a bankers briefcase and copied the info and attempted to admit it as evidence

against 3rd party. The trial judge thought it was outrageous and did not allow the evidence.a. S.C. Held: the supervisory power does not authorize a federal court to exclude evidence

that did not violate the defendant’s 4A rights [since it was the banker’s rights who were violated, not Payner’s].

i. Problem: takes admittedly illegal behavior and immunizes it.

III. HARMLESS ERROR SITUATIONS

1. United States v. Hasting (1983); Five D’s were convicted of kidnapping and transporting V across state lines. Prosecutor commented on failure of any D’s to take the stand, and CoA reversed.

a. S.C. Held: that although impermissible comment on a D’s failure to take the stand was harmless error, the supervisory power may not be asserted simply to justify a reversal of criminal convictions. Convictions stand.

3. TRENDS AND COUNTERTRENDS: THE “NEW FEDERALISM IN CRIMINAL PROCEDURE” AND NEW LIMITATIONS ON STATE RIGHTS PROTECTIONS

a. CAN A STATE CONSTITUTION BE MORE D FRIENDLY THAN FEDERAL CONSTITUTION? i. Yes. But you would want to raise state issues parallel w/ federal issues.

ii. A state supreme court bent on providing the accused w/ greater protection that that said to be required by the federal constitution must be careful to make it clear that it is resting its ruling on an independent state ground.

iii. State court must indicate “clearly and expressly” that it’s ruling is based on separate, adequate and independent” state grounds.

iv. U.S. v. Robinson (2008); S.C. held that a full search is permissible after a lawful custodial arrest, regardless of the crime giving rise to the arrest.

1. But…W/in 4 years, 4 different state courts had held that the nature of the offense is relevant to whether a full search is justified.

4. DUE PROCESS, INDIVIDUAL RIGHTS AND THE WAR ON TERRORISM: WHAT PROCESS IS CONSTITUTIONALLY DUE A CITIZEN WHO DISPUTES HIS ENEMY – COMBATANT DESIGNATION?

A. Hamdi v. Rumsfeld (2004); D is U.S. citizen who is arrested in Afghanistan and turned over to the U.S. military as a Taliban fighter. Gov’t detained D, saying he was “enemy combatant,” NO OPNION of S.C.

i. HELD: a citizen-detainee seeking to challenge his classification as an enemy combatant must receive notice of the factual basis for his classification, and a fair opportunity to rebut the gov’ts factual assertions b/f a neutral decisionmaker.

b. Rasul v. Bush (2004); S.C. ruled that U.S. courts have jurisdiction to consider challenges to the legality of the detentions of foreign nationals captured abroad and imprisoned at Guantanamo Bay.

C. Hamdan v. Rumsfeld (2006); U.S. Prez issued the November 13 Order, which gave the Sec. of Defense the authority to appoint military commissions. D captured in Afgh., and transported to G-Bay, and was subject to trial by military commission.

i. HELD: b/c its structure and procedure violated both the Uniform Code of Criminal Justice and the Geneva Conventions, the military commission lacked power to proceed.

d. Boumediene v. Bush (2008); Foreign war criminals held in G-Bay. How long can you hold them b/f you try them? The issue was whether the detainees had the constitutional privilege of habeas corpus.

i. HELD: Ds were entitled to H.C. to challenge their detentions as the Military Commissions Act did not formally suspend the writ of H.C. In order to deny habeas corpus, Congress had to act in conformance with the Suspension Clause. We have to hear them, and it has to be in a court (not military tribunal).

3

Page 4: Crim Pro Outline 1.1

5. Right to Counsel, “By Far the Most Pervasive” Right of the Accused; Equality and the Adversary System

A. THE CONCEPT OF POVERTY

i. For criminal justice purposes, ‘poverty’ is relative to the needs of the accused, and arises when, at any stage of the proceedings, lack of means prevents the accused from properly asserting a right.

B. THE OBLIGATION OF EQUAL JUSTICE

i. The purpose of the government’s obligation to provide ‘equal justice’ is not to eliminate discrimination against the poor, but to preserve the integrity of the justice system by preventing poverty from impeding the proper functioning of that system

C. POVERTY AND THE ADVERSARY SYSTEM

i. The essence of the adversary system is challenge.ii. Because our justice system is adversarial, the defense must function properly for the system itself to

function properly

d. SYSTEMS USED TO PROVIDE COUNSEL FOR THE INDIGENT; INDIGENCY STANDARDS i. State Delivery System: Individually appointed private attys and Public Defenders Offices.

E. REPAYING COST OF DEFENSE

i. A state may require, as a condition of probation, that a convicted defendant repay the cost of his defense if his indigence ends.

ii. Fuller v. Oregon (1974); S.C. held that such recoupment statutes do not violate equal protection, finding the distinction between those convicted and those acquitted ‘wholly non-invidous.’

1. The Court further held that the possibility of a repayment obligation does not ‘chill’ the right to counsel.

iii. NOTE The ABA disagrees as to the danger of chilling the right to counsel, and recommends ordering repayment only where the claim of indigence was fraudulent.

F. THE RIGHT TO APPOINTED COUNSEL AND RELATED PROBLEMS

i. Early in our country’s history, the 6A only granted criminal Ds the right to be represented by a lawyer, but not to have the government provide one. Gradually, a broader interpretation of the right to counsel began to develop

ii. State Capital Cases; 1932 S.C. recognized a right to appointed counsel in STATE capital cases, but focused on the ‘fundamental fairness’ due process approach, rather than 6A rights; Powell v. Alabama

iii. Federal Defendants; 1938, S.C. held that the 6A guarantees federal D a right to appointed counsel.

IV. STATE DEFENDANTS

1. Betts v. Brady (1942); D, an indigent, was indicted for robbery. D’s request for counsel was denied b/c local practice permitted appointment only in rape/murder. D convicted.

a. Rule: Federal courts must appoint counsel for ALL federal crimes, but state courts need NOT appoint attorneys for all indigent criminal defendants.

2. Gideon v. Wainwright (1963); Indigent represented himself in a burglary trial, after requesting assistance of counsel, and was convicted. Overrules Betts.

a. Rule: The 6A’s right to counsel, through the 14A, REQUIRES States to appoint counsel for indigents in felony cases.

b. Rationale: In our adversary system of criminal justice, any person haled into court, who is too poor to hire an atty, can’t be assured a fair trial unless atty is provided for him.

V. NO LAWYER = NO PRISON SENTENCE

1. Argersinger v. Hamlin (1972); Absent a knowing and intelligible waiver, no person may be imprisoned for any offense unless he was represented by counsel.

2. However, prison sentence must occur for this right to attacha. Scott v. Illinois (1979); D charged w/ shoplifting and denied atty. The 6A and 14A

require ONLY that NO indigent criminal D be SENTENCED to a term of imprisonment unless the State has afforded him the right to assistance of appointed counsel in his defense.

3. Enhancement of Subsequent prison sentencea. Nichols v. United States (1994); An uncounseled conviction, valid because it did not

result in imprisonment for that offense, may yet form the basis for enhancing a prison sentence for a subsequent offense.

4. Suspended Sentences or Probationa. Alabama v. Shelton (2002); D given no atty. Court suspended sentence and gave D two

years probation on condition that nothing is done in the future. Then D committed another crime and court wishes to impose the suspended sentence.

4

Page 5: Crim Pro Outline 1.1

i. Issue: Where the State provides no counsel to an indigent D, does the 6A permit activation of a suspended sentence upon the D’s violation of the terms of probation?

ii. RULE: S.C. says that this is not allowed since the first crime was convicted without counsel being appointed. A D who receives a suspended or probated sentence TO IMPRISONMENT has a constitutional right to counsel.

VI. WHEN THE RIGHT TO COUNSEL BEGINS

1. When else does D get a lawyer? Any time the judge might put D in jail!

2. RULE: A person is entitled to the help of a lawyer – assuming the stage of the prosecution is a “critical one” – at or after the time that judicial proceedings have been initiated against him – whether by way of formal charge, preliminary hearing, indictment, information, or arraignment (presumably as early as the 1st appearance b/f a judicial officer).

a. Adversary judicial proceedings have commenced (6th A right to counsel is guaranteed only in all criminal prosecutions) and,

b. The encounter is a critical stage of the criminal proceedings. i. No right to atty in lineup or photo ID stage b/c not critical.

3. Rothgery v. Gillespie County (2008); D in Texas denied counsel until entry of information or indictment per TX statute. S.C. vacated judgment and remanded.

a. RULE: You have a right to atty when you are brought b/f judge and told “this is what you are charged with.” Right to counsel starts at the initial appearance.

G. THE GRIFFIN-DOUGLAS “EQUALITY” PRINCIPLE:

”THERE CAN BE NO EQUAL JUSTICE WHERE THE KIND OF TRIAL A MAN GETS DEPENDS ON THE MONEY HE HASi. Griffin v. Illinois (1956); In Illinois, an appeal usually required a transcript of the trial to meet all of the

procedural requirements. D was financially unable to purchase the transcript to make his appeal.1. Rule: State cannot require indigent D to pay filing fee b/f permitting him to appeal. Indigent D

must be furnished free transcript of HC hearing for use on appeal. Indigent D entitled to free transcript of preliminary hearing for use on appeal.

II. IMPACT OF PRINCIPAL

1. State cannot require indigent defendant to pay FILING FEE before permitting him to appeal; 2. Extending ban on filing fees to state post-conviction proceedings;3. Indigent must be furnished a free transcript of a state habeas corpus hearing for use on appeal from

a denial of habeas; 4. State must provide adequate transcripts to indigent defendants even where they are subject only to

fines and not to imprisonment; Mayer v. Chicago (1971)a. Rationale: The price of the defendant’s pocketbook bears no more relationship to his

guilt or innocence in a non-felony than in a felony case.

III. RIGHT OF APPEAL CASES

1. Douglas v. California (1963); S.C. held that a procedure like the one used by the state appellate court in which an indigent D was denied counsel on appeal unless he first made a preliminary showing of merit did not comport with fair procedure. In vacating the judgment of the state appellate court, S.C. stated that where the merits of the one and only appeal an indigent had as of right were decided without benefit of counsel, an unconstitutional line had been drawn between rich and poor.

IV. DISCRETIONARY APPEALS

1. Ross v. Moffitt (1974); An indigent convicted forger challenged a state policy of not providing counsel for discretionary appeals to the state supreme court or certiorari petitions to the federal Supreme Court. S.C. found that neither DPC nor the EPC required the state, after furnishing counsel for indigent’s 1st appeal as of right from conviction, had to appoint counsel for the indigent’s subsequent discretionary appeals.

a. Rule: State must provide transcripts and atty for 1st appeal as of right, but no more.b. Rationale: An indigent’s grade of justice must meet minimum standards of fairness to

satisfy due process, but it need NOT be the SAME grade of justice that a wealthy man can buy.

c. Remember… the only issues that can go up are issues that atty raised the 1st time around, so it’s easier for the courts to deny subsequent review. BUT…S.C. says they will decide who gets atty after 1st appeal based on the record and brief.

5

Page 6: Crim Pro Outline 1.1

V. EXPERT SERVICES

1. D will want own DNA expert, forensic expert, ballistic expert, psychiatric expert - don’t want to trust the states’ guys – they are already saying you are guilty.

2. Ake v. Oklahoma (1985); Dealt with an indigent’s right to have a psychiatric evaluation provided by the state.

a. RULE: State must provide D with the “BASIC TOOLS” to ensure meaningful access to justice. This has come to mean not just something fundamental to a defendant’s legal arsenal, but a resource without which the defense fails.

3. Psychiatric Expert only if sanity is significant issue, or if state present “future dangerousness” evidence is capital case.

6. WITHDRAWAL OF APPOINTED COUNSEL ON APPEAL: THE POTENTIONAL TENSION B/TW THE INGIDENT D’S RIGHT TO COUNSEL ON 1ST APPEAL OF RIGHT AND THE LAWYER’S ETHICAL OBLIGATION NOT TO ASSERT FRIVOLOUS CLAIMS

a. Steps for an Appeal: i. Ask for a transcript,

ii. Look for sufficient evidence, objections on the records, whether jury instructions accurately state the law, LOOK FOR ERRORS that might lead to reversal.

b. Ander v. California (1967); If, after a “conscientious examination” of indigent D’s case, court appointed atty finds an appeal to be “wholly frivolous,” he should advise the court and request permission to w/draw.

i. Anders brief: Where atty writes to the court that they don’t think there is anything to the appeal, but also told to point out any argument in D’s favor (makes no sense). There is always something to argue, you could always take the point that the law should change.

7. THE RIGHT TO APPOINTED ATTY IN PROCEEDINGS OTHER THAN CRIMINAL PROSECUTIONS

A. PROBATION AND PAROLE REVOCATION HEARINGS; JUVENILE COURT PROCEEDINGS; PARENTAL PROCEEDINGS

i. Gagnon v. Scarpelli (1973); S.C. held that an indigent probationer or parolee has no unqualified right to be represented by atty at revocation hearing.

ii. Mempa v. Rhay ; S.C. held a probationer is entitled to be represented by atty at a combined revocation and sentencing hearing b/c sentencing is a stage of criminal proceeding where substantial rights of D affected.

iii. Others:1. Probation hearing NO

a. But maybe if request for atty made on timely and credible claim that D didn’t commit violation or there was substantial mitigating reason for violation.

2. Combined Revocation and Sentencing Hearing YESa. Only if there are fact sensitive issue as to whether or not violation has occurred

3. Juvenile Proceedings that may lead to imprisonment YES (even though these are not technically criminal proceedings)

4. Summary Court-Martial NO Major Court-Martial Probably5. Parental rights permanently revoked YES, Equate loss of child as a loss of liberty

iv. Turner v. Rogers ; Child support non-payment case, S.C. held that where a custodial parent (one entitled to child support) is unrepresented by atty, the state need not provide atty for non-custodial parent.

1. But, D needs to find out if indictment for non-payment is; a. Criminal (he could go to jail) - he gets atty, or b. Civil (meant to coerce compliance w/ order – but Congress said you can only be held in

jail on civil contempt for so long) – no atty.

8. THE ROLE OF COUNSEL

A. WAIVER OF THE RIGHT TO COUNSEL; THE RIGHT TO PROCEED PRO SE

i. Faretta v. California (1975) D did not want to be represented by public defender. D had represented himself b/f, had H.S. education. Judge questioned D on hearsay rule and found D had no right to represent himself; judge appointed public defender.

1. RULE: The 6A guarantees D in a State criminal trial has an independent constitutional right of self-representation and that he may proceed to defend himself without counsel when he VOLUNTARILY and INTELLIGENTLY elects to do so. He should be made aware of the dangers and disadvantages of self-representation, so that the record will establish that “he knows what he is doing and his choice is made with eyes open.”

2. Rules: a. Anyone who wants to self-represent has a constitutional right to do so.

6

Page 7: Crim Pro Outline 1.1

b. D does not have a constitutional right to the assistance of standby atty. i. Hybrid Representation: D and atty share responsibility for trying the case.

Preferable for D w/ serious charge b/c D will need to humanize himself & one way to do this is to have jury get to know D by having D participate in some way, voir dire, etc. Prosecution will want appellate proof trial, so will prefer hybrid over pure pro se.

c. D’s rights are NOT violated when judge appoints standby atty – even over D’s objections – to relieve the judge of the need to explain and enforce basic rule.

i. Limitations on Standby: Pro se D is allowed to control his own trial and atty can’t undermine D’s authority.

3. Exception: No right to self-representation on appeal.

ii. Indiana v. Edwards (2008); D competent to stand trial (Test: D was able to assist counsel in his defense), but he was not competent to defend himself. After conviction, D asked for new trial b/c he was denied right to self-represent. The S.C. found the Constitution permitted the State to limit D’s self-representation right by insisting upon representation by atty at trial on the ground that he lacked the mental capacity to conduct his trial defense.

1. Rule: The Constitution does not permit trial of a D who lacks “mental competency.” The competency standard is defined as including both:

a. Whether the D has a rational as well as factual understanding of the proceedings against him, and

b. Whether the D has sufficient present ability to consult w/ his atty w/ a reasonable degree of rationale understanding.

2. Rule: There is a point where a person is so mentally ill that we will not permit them to represent themselves. Problem: the standard used depends on the trial judge.

B. THE CONSTITUTIONAL DIVISION OF AUTHORITY OVER DEFENSE DECISIONS

i. Jones v. Barnes (1983); S.C. held that appellate atty did not have to present a non-frivolous claim that her client wished to press if atty believed that the better strategy was to limit her argument and brief to other issues. It is for the atty to decide which claims are strong enough to be presented consistent w/ strategy, but if atty gets it wrong, the issues not brought up are waived.

ii. D decides whether to take each of the following steps: 1. Plead guilty or take action tantamount to entering a guilty plea; 2. Whether or not to accept a bargain (atty has obligation to present bargain to client); 3. Personal decisions; 4. Waive the right to jury trial; 5. Waive his right to be present at trial; 6. Testify on his own behalf; 7. Forgo an appeal.

iii. Rationale: these decisions involve “fundamental rights”

iv. Atty has the ultimate authority in deciding whether or not to advance the following defense rights: 1. Strategic decisions; 2. Decisions involving matters requiring the “superior ability of trained counsel” in assessing

“strategy.” 3. The Gonzalez Ruling (2008): The defense consent needed by statute to allow a federal magistrate

to conduct voir dire and jury selection is satisfied by the express consent of counsel alone.

C. ATTY/CLIENT DISAGREEMENT AND THE REPLACEMENT OF COUNSEL

i. Indigent D has a right to substitution of new atty upon showing of “good cause,” but an atty’s refusal to accept D’s direction on strategy, does not constitute “good cause.”

ii. Good Cause: created by a conflict of interest or a complete breakdown in communication, and the latter can’t have been the product of D’s abusive or uncooperative behavior.

iii. Lack of confidence in one’s counsel does not constitute good cause, nor does lack of meaningful relationship between the two. Morris v. Slappy

D. NO PRESUMPTION OF WAIVER OF RIGHT TO COUNSEL

i. There is no presumption of waiver of counsel; rather there is a strong presumption AGAINST it;1. “The record must show, or there must be an allegation and evidence which shows, that an accused

was offered counsel but intelligently and knowingly rejected the offer. Anything less is not waiver.” Carnley v. Cochran

7

Page 8: Crim Pro Outline 1.1

ii. A judge must investigate as long and as thoroughly as the circumstances of the case before him demand whether there has been an effective waiver of counsel;

1. “The fact that the accused may tell him that he is informed of his right to counsel and desires to waive this right does not automatically end the judge’s responsibility.” Von Moltke v. Gillies

iii. Appeal1. There is NO right to self-representation at appellate proceedings. Martinez

iv. Informed Right to Pro Se?1. Most courts have held that a D need not be informed of his right to represent himself

v. Asserting the right1. If not timely asserted, the right to self-representation may be waived

E. RIGHT TO “EFFECTIVE” COUNSEL

i. GENERALLY: There is a constitutional right to EFFECTIVE assistance of counsel. Powell v. Alabama1. Public defenders routinely spend very little time with their clients (30% < 10 min., 32% < 10-20

min., 20% < 30-180 min., 50% of D’s thought atty appointed worked for state)2. There is a constitutional right to effective assistance of counsel on an APPEAL OF RIGHT.

a. However, in proceedings where there is no right to appointed counsel—such as a writ of certiorari to a state supreme court—the right to effective assistance of counsel does not apply. D must “bear the consequences of his unwise choice.” Wainwright v. Torna

II. TEST FOR EFFECTIVE COUNSEL: STRICKLAND TEST

1. Strickland v. Washington (1984) At the sentencing phase, D’s atty failed to investigate any character witnesses as mitigating factors. D was sentenced to death after pleading guilty to capital offenses, and thereafter sought reversal of his conviction based upon ineffectiveness of counsel.

2. BENCHMARK

a. Whether counsel’s conduct so undermined the proper functioning of the adversarial process that the trial cannot be relied on as having to produce a just result.

b. When judging the performance of counsel, we are to give deference to the tactical or strategic decisions of the lawyer. Can’t find deficiency if we later say the strategy was bad (hindsight)

i. How do you get something like this reviewed? Problem – no case is tried perfect. How do you show-prove deficiency? Not easy b/c atty has to be self-reflective.

1. Bluth tells client that maybe they would want a fresh set of eyes to look at this on appeal to see any issues. Hard on appeal b/c facts are fixed in transcript, and many of the deficiencies are OMISSIONS and won’t be on the transcript. Maybe do a post conviction fact hearing.

2. Problem today is if the appeal is heard in the state supreme court, it is not reviewable in federal court on HC, or the state S.C. is to be given deference. This means the federal court is not to disturb the state S.C. opinion unless unreasonable. Need a better record earlier

3. STRICKLAND TEST WHAT THE DEFENDANT MUST SHOW:a. 1st Prong Deficiency: Did the atty’s performance fall below the standard reasonably

expected of other attys in the community (malpractice); ANDi. Standard: that of a reasonably effective assistance

ii. Problem: Should the standard be local or national?b. 2nd Prong Prejudice: Did the deficient performance prejudice the defense? (But for

the malpractice, the outcome would have been different.)i. Problem: How do you prove this? How can you prove what a jury would

have done? You can’t make this determination easily.1. The question is whether there is a reasonable probability that, absent

the errors, the fact-finder would have had a reasonable doubt respecting guilt.

4. Example: 9th Circuit held ineffective assistance of counsel in a murder prosecution when counsel had not consulted forensic blood experts in developing the basic strategy for D’s defense or offered their testimony as part of the principal case for the defense.

5. BASIC STANDARDS OF PRACTICE PER 6TH CIRCUIT: a. Needs to investigate each claim; b. Read the statute for the elements and penalty; c. Interview the client – what do they have to say about what happened;

8

Page 9: Crim Pro Outline 1.1

d. Talk to witnesses; e. Do discovery on prosecutors file; f. Look to see if any motions need to be filed (claims to suppress evidence, etc.); g. Are there time limits you need to follow; h. Need to know the law – do legal research; i. Come up w/ trial strategy; j. Communicate w/ client if any offers are made.

III. APPLICATION OF STRICKLAND

1. THE ROLE OF THE STANDARDS OF PROFESSIONAL RESPONSIBILITY

a. Nix v. Whiteside ; (1986) Nix (D) claimed ineffective assistance of counsel because his attorney said that he would report Nix if he committed perjury during trial. Client ended up not testifying the way that he had wanted to. He then said that he had ineffective assistance of counsel b/c the lawyer refused to let him testify the way he wanted.

i. Analysis: Was the representation deficient? Courts say no, can’t let client on the stand to lie.

ii. HELD: The 6A right to assistance of counsel is not violated when the D’s atty refuses to cooperate in the D’s presenting perjured testimony. There is no constitutional right to testify FALSELY.

iii. What are you supposed to do if you KNOW that the client is lying?1. Withdrawal from representation, if possible,

a. Judge won’t want to start over, you will ask D open ended question “tell jury what happened” and then ignore it.

2. Do not treat the testimony as actually happening, 3. Do not refer to it in closing.

iv. Both sides in criminal cases have to deal with situations of improbable and possibly false testimony.

2. THE CLASSICAL “STRATEGIC DECISION” a. Yarborough v. Gentry ; D’s atty gave brief closing argument and D claimed ineffective

counsel. S.C. Ruled the right to effective assistance extends to closing arguments. Nonetheless, counsel has wide latitude in deciding how best to represent the client, and deference to counsel’s tactical decision in closing presentation is important b/c of the broad range of legitimate defense strategies at this stage.

b. State v. Davis ; atty told jury he was a racist, but they couldn’t let this be a factor.i. S.C. said this is not a legitimate tactical approach.

3. DUTY TO INVESTIGATE IN CAPITAL CASES a. Wiggins v. Smith ; state court said atty had made a reasonable strategic decision not to

present any evidence of D’s personal history at sentencing. S.C. found this unreasonable in light of the inadequacy of the investigation that led to atty’s decision. RULE: Atty must investigate personal history and present evidence at sentencing, unless D refuses to allow the presentation of the evidence.

b. Rompilla v. Beard (2005); D found guilty of murder and given death. D claimed ineffective assistance b/c trial atty failed to present mitigating evidence about D’s childhood, mental capacity, health and alcoholism. If you know state is going to rely on files you need to look at them as well. Performance was deficient (per Bluth)- Problem is how do you show that D wouldn’t have been sentenced to death if Atty had presented this info.

i. RULE: D’s atty is bound to make reasonable efforts to obtain and review material that counsel knows that prosecution will probably rely on as evidence of aggravation at the sentencing phase of trial. Atty’s job in penalty phase is to save D’s life.

c. Schriro v. Landrigan (2007); Not unreasonable for court to conclude that a D who refused to allow the presentation of any mitigating evidence could not establish Strickland prejudice based on his atty’s failure to investigate further possible mitigating evidence.

4. THE CONSTRUCTIVE DENIAL OF COUNSEL DOCTRINE a. Bell v. Cone (2002); Where counsel’s absence at a critical proceeding or lack of effort

was so extensive as to produce a “complete failure” of representation, a presumption of prejudice is justified.

9

Page 10: Crim Pro Outline 1.1

b. Burdine v. Johnson (2001); majority refuses to adopt per se rule that any sleeping by defense atty during trial merits presumption of prejudice. (Bad rule per Bluth) Atty must be asleep throughout the trial, not just intermittently.

F. CONFLICTS OF INTEREST AND MULTIPLE REPRESENTATION

i. Generally: The representation of multiple defendants by the same lawyer presents an opportunity for conflicts of interest (e.g., where it is in the best interest of one defendant to act against the best interest of the others) at all stages in the proceedings. Atty can’t advise a client to testify against the other or to not take a deal.

1. ABA Model Rule 1.7: Atty must get off of one of the clients case, can’t represent them both b/c someone will be offered something from prosecuting that will make it impossible (although code says it might be OK, it won’t).

II. DEFINING THE SCOPE OF THE DUTY TO INQUIRE INTO CONFLICTS OF INTEREST

1. Cuyler v. Sullivan (1980) D and his two co-Ds were represented by the same attys in separate trials, and after D was convicted and the other two acquitted, he claimed denial of effective counsel due to conflict of interest.

a. Rule: The mere possibility of a conflict of interest does NOT deprive a D of his right to counsel; rather a D must establish that an actual conflict of interest adversely affected his lawyer’s performance in order to prove a violation of his 6A rights.

i. 2 Prongs:1. An ACTUAL conflict existed; and2. The conflict actually cause the attorney to act ADVERSELY to the

defendant’s interests. b. Standard: The trial judge does not have an affirmative duty to inquire about a lawyer’s

potential conflict of interest UNLESS the trial court knows or reasonably should know that it exists. Bluth says this has changed and judge must now inquire.

III. POST CONVICTION REVIEW

1. General Rule: A D alleging a 6A violation must demonstrate “a reasonable probability that, but for counsel’s unprofessional errors, the result of the proceeding would have been different.”

2. Mickens v. Taylor ; (2002) Mickens was convicted of murder and rape and sentenced to death; the atty who prepared his habeas petition found that his trial atty had been representing the V at the time of the murder.

a. Rule: Until a D shows that his counsel actively represented conflicting interests, he has not established the constitutional predicate for his claim of ineffective assistant. Must have an actual conflict.

b. Problem: How do you ever prove that the outcome was affected?

3. Example: You are small town atty, and one of the witnesses against your client was a former client of yours. You know witness has a violent temper, and is out to get your client – info told in confidence and can’t be used. So you can’t defend your client b/c you are holding back info. Can’t take the case b/c you will be conflicted counsel.

4. Hypo: Prosecutors in small counties often do civil cases, and often a jury member will be a former client. What to do? Ask the jury what they thought of the prosecutor as their atty.

G. DISQUALIFICATION OF COUNSEL

i. Wheat v. United States (1988); Wheat’s (D) motion to substitute in the counsel of choice was denied, in spite of waiver, because of potential conflict of interest. Gov’t wouldn’t give D atty he wanted – this is not good 6A jurisprudence. Would have been separate trials, and D’s would have used 5A and not testified, therefore not implicated each other.

1. Rule: The court is allowed substantial latitude in refusing waivers of conflicts of interest with respect to a criminal D’s chosen counsel both in cases where there is an actual conflict and in those where a serious potential for conflict exists.

2. Note: A situation like this gives the prosecutor an awful lot of power because he could use the conflict of interest to remove available lawyers from representing the accused.

3. What conflicts would disqualify defense atty? a. If Atty represented gov’t witness who is involved in his current case (ex: atty represented

cop in bribery case, now cop is chief witness in D’s case) Atty must disclose the relationship and outcome should be up to D, but WHEAT Court says this is up to the Judge and a motion by the gov’t to disqualify.

10

Page 11: Crim Pro Outline 1.1

b. If D participated in joint defense agreement. We allow criminal D’s to get together and form joint conference where everything said cannot be used against anyone. Not waiver of 5A.

c. If D’s atty is a witness – must get off case. d. If D’s atty involved in crime – off of case.

H. THE RIGHT OF “COUNSEL OF CHOICE”

i. General Rule: An indigent D does not have the right to appointed counsel of choice, leaving the selection of counsel up to the discretion of the trial judge.

ii. Rationale: Judges assume that they can choose a more able attorney than the indigent because they know the abilities of the available local counsel.

iii. Admission Pro Hac Vice: If Bluth wants to go to NJ and try a case, can he if someone is willing to pay him?

1. Federal Court: Will let anyone in who is licensed in another state, but may require local counsel to assist. States are a little stingier.

2. State Courts: Larry Flynt charged w/ selling obscene mags in Cincy, trial judge did not allow out of state 1st A atty w/ local counsel to try Flynt’s case. Ohio courts upheld this decision.

iv. United States v. Gonzalez-Lopez (2006); Missouri pot case. D’s family hired Fahle, D hired Low (CA Atty). Court rejected D’s choice (b/c he passed a note to another atty – no such rule per Bluth) and appointed Dickhaus, Low had to sit in audience and could not defend D. D convicted and claims trial court deprived him of choice of counsel. Gov’t argues they can’t prove second prong of Strickland, but this has nothing to do w/ Strickland. Held: D has right to choose counsel, and it was violated. This has nothing to do w/ unfair trial, pure right to chose.

1. Rule: 6A guarantees the D the right to be represented by an otherwise qualified atty whom that D can afford to hire, or who is willing to represent the D even though D w/o funds.

v. REPLACING APPOINTED COUNSEL

1. Morris v. Slappy ; D had good relationship w/ appointed Atty, who got sick b/f trial. D asked for continuance, Judge denied and gave D new atty (3 weeks b/f trial). If this was a privately paid atty, judge would have agreed.

a. Rule: S.C. said lack of confidence or of a “meaningful relationship” with indigent’s counsel does not constitute good cause, nor does lack of meaningful relationship between the two.

2. Example: Bill and Emily Harris kidnapped Patty Hearst. Over the course of years they had established relationship w/ like-minded atty. When Bill and Emily were arrested for serious armed robberies atty said he was available and would represent, he just needed court to appoint him as counsel. Judge said NO – judge gets to choose appointed counsel, and appointed new/young atty.

a. Rule: Judges have to have a valid reason to say NO to an appointment; here, judge had no reason to say no. Atty license, available and willing.

VI. DEFENDANT’S ACCESS TO FUNDS

1. Caplin; (1989) D thought he might be charged w/ selling coke and retained atty, paid retainer up front. D pays atty $25K cash. Pursuant to a statute authorizing forfeiture of assets, the federal government froze D’s assets and rendered him unable to pay for the atty he had retained to defend him in the criminal proceedings. The Court holds that the government’s interest in collecting forfeitable assets outweighs any 6A right a D has to use such assets for his defense.

a. Problem 1: Over $10K in cash must be reported to IRS. Can’t advise client how to get around this requirement (structuring). If client doesn’t want it reported, atty must fill out the form and leave client info blank (and hope IRS doesn’t come after atty).

b. Problem 2: Gov’t is going to say this is money gained through illegal activity, and belongs to gov’t – will insist atty turns over money (Asset Forfeiture).

c. Rule: The forfeiture statute does not impermissibly burden a defendant’s 6A right to retain counsel of choice, OK for gov’t to collect funds gained through illegal activity.

d. Difficulty of Case:i. On the one hand it seems perfectly logical to deny a criminal D access to ill-

gotten gains in order to fund his defense; yet it seems unfair to allow the government to render a D—who has not been found guilty—indigent, and deprive him of his constitutional right to choose his atty. Allows gov’t to freeze private atty’s out of large part of crim market. Gives gov’t wide

11

Page 12: Crim Pro Outline 1.1

breadth, if they get the money 1st, D gets public atty b/c atty can’t take criminal case on a contingent fee basis (Rules of Professional Conduct).

e. Solution: Atty must find out where the fee is coming from – is there a legitimate source of income for atty’s fees. Find out or you are working for free.

9. 4TH AMENDMENT - ARREST, SEARCH AND SEIZURE

A. THE EXCLUSIONARY RULE – THE 4TH AMENDMENT

i. Search and seizure clause: “The right of the people to be secure in their persons, houses, papers, and effects, against unreasonable searches and seizures, shall not be violated . . .”

ii. Warrant clause: “[A]nd no Warrants shall issue, but upon probable cause, supported by Oath or affirmation, and particularly describing the place to be searched, and the persons or things to be seized.”

iii. Rationale: Colonist were constantly searched by British using “general warrants” to search anyone’s house w/o probable cause. Interposes a judge/magistrate in between government and person sought to be searched.

iv. Exclusionary Rule: Evidence obtained in violation of the 4A may NOT be used against the individual whose rights were violated in obtaining the evidence.

1. Purpose of Exclusionary Rule is to impeded/deter police from violating the 4A. 2. Applies equally to state and federal courts per Mapp v. Ohio.

v. Exclusionary Good Faith Exception: So long as the police have a good-faith belief that a warrant has been properly issued by a magistrate and based on sufficient probable cause, evidence obtained pursuant to the warrant is admissible.

1. Rationale: The marginal or nonexistent benefits produced by suppressing evidence obtained in objectively reasonable reliance on a subsequently invalidated search warrant cannot justify the substantial cost of exclusion. Reliance on faulty warrant must be objectively reasonable.

2. TEST: Would a reasonable cop have known this affidavit was based upon insufficient information? What should a reasonable cop know? 1. The place to be searched, and 2. The things to be seized.

vi. How do you enforce this rule? Bad rule in the sense that it lets criminals go free. On the other hand, if we don’t let the criminal go free are the cops ever going to enforce 4A values.

B. EXCLUSIONARY RULE CASES:

i. Weeks – Original exclusionary case as applicable to federal casesWolf – Said that states didn’t need to apply exclusionary rule

ii. Mapp v. Ohio (1961)1. Facts: Mapp’s home was forcibly searched w/o a search warrant. Mapp seeks to have the

evidence seized therein suppressed under the 4A’s exclusionary rule.2. Rule: The exclusionary rules requiring evidence gathered in violation of the 4A to be excluded

from criminal proceedings applies equally to both state and federal governments. 3. Courts don’t like the exclusionary rule, but there is no alternative to enforce 4A values.

C. GOOD FAITH EXCEPTION TO EXCLUSIONARY RULE

i. United States v. Leon (1984); Police conducted a search using a warrant they believed in good-faith to be based on sufficient probable cause, but which later found to be lacking in that regard.

1. Rule: So long as the police have a good-faith belief that a warrant has been properly issued by a magistrate and based on sufficient probable cause, evidence obtained pursuant to the warrant is admissible.

2. Rationale: The marginal or nonexistent benefits produced by suppressing evidence obtained in objectively reasonable reliance on a subsequently invalidated search warrant cannot justify the substantial cost of exclusion.

a. The officer’s reliance on the magistrate’s probable-cause determination and on the technical sufficiency of the warrant he issues must be objectively reasonable, and it is clear that in some circumstances the officer will have no reasonable grounds for believing that the warrant was properly issued.

12

Page 13: Crim Pro Outline 1.1

b. There can be situations where the warrant would be objectively unreasonable. In the absence of an allegation that the magistrate abandoned his detached and neutral role, suppression is appropriate only if the officers were dishonest or reckless in preparing their affidavit or could not have harbored an objectively reasonable belief in the existence of probable cause.

c. Normally if the judge screws up we have a re-trial. Here, we don’t have this opportunity, can’t give back the evidence. If officers are going to search, they need to go to judge (neutral and detached person) to determine PC, if judge finds PC cops can go search. Cops did nothing wrong in this, so we are not going to suppress the evidence.

d. TEST: Would a reasonable cop have known this affidavit was based upon insufficient information? What should a reasonable cop know? The place to be searched, and the things to be seized.

e. Remember, the 4th A applies to BOTH searches and seizures, and they can only be done on PC (not suspicion!).

3. Judges rubber stamp police affidavits all the time.

D. KNOCK-AND-ANNOUNCE:i. Hudson v. Michigan (2006); Cops executing a warrant for drugs and firearms at Hudson’s home

announced their presence, but waited only “3-5 seconds” b/f entering. Hudson moved to suppress the drugs and weapon found, arguing that the premature entry violated his 4A rights.

1. RULE: Suppressing evidence for knock-and-announce violations is unjustified. 2. Dissent: this gives police no reason to comply w/ warrant and knock-and-announce rules b/c the

evidence won’t be suppressed. 3. Rationale: Knock-and-announce protects both cops and civilians. If there is no way of

deterring behavior, are you ever going to get enforcement?

E. LIMITATIONS TO THE SCOPE OF THE EXCLUSIONARY RULE

i. Retroactive? NO 1. In the 1980’s cops stopped car for speeding, took 4 passengers out of car and searched car. Found

jacket w/ pot. D’s argued how you can do this search after a speeding ticket. This case was reversed – now what happens to other cases in pipeline? They will be reversed as well. Belton

2. Davis v. U.S . (2011); routine traffic stop, cop searched car and found gun in Davis’ jacket. This was an allowable search per precedent, but then the precedent for the search was later overruled (Belton). S.C. held that searches conducted in objectively reasonable reliance on binding appellate precedent are not subject to the exclusionary rule.

a. Remember, if the purpose of the exclusionary rule is to deter police misconduct, there is no deterrent effect here b/c cops followed the rules as they were at the time.

ii. Grand Jury Proceedings NO1. The exclusionary rule does not apply to grand jury proceedings because this would interfere with

the grand jury’s duties and would not further the goal of deterring police misconduct. Grand jury witnesses therefore may not refuse to answer questions on the ground that they are based on illegally obtained evidence. United States v. Calandra (page 248)

iii. Evidence Obtained and Used in Criminal Case After Conviction (for consideration by judge in determining the sentence?) Yes, they can use the evidence that was suppressed at trial for sentencing b/c no appreciable increment in deterrence would result from applying a 2nd exclusion at sentencing after the rule has been applied at the trial itself. U.S. v. Schipani

iv. Parole Revocation Proceedings NO1. The S.C. has declined to apply the exclusionary rule outside the criminal trial context, to parole

revocation proceedings, because it would provide minimal deterrence benefits and would hinder the functioning of state parole systems. PA Bd. of Probation and Parole v. Scott

v. Civil Forfeiture Proceedings YES1. The 4th A exclusionary rule DOES apply to civil forfeiture proceedings because they are “quasi-

criminal,” so if the item is seized illegally, it can’t be forefeited – especially if it’s not contraband. One 1958 Plymouth Sedan v. Pennsylvania

2. HOWEVER, where officers illegally obtain evidence for use in a criminal trial, the exclusionary rule does NOT bar admission of that evidence in a civil suit involving a different sovereign (i.e., the federal government (IRS) rather than the state) because this would not further the goal of deterring the officers’ misconduct. United States v. Janis

vi. Civil Deportation Proceedings NO

13

Page 14: Crim Pro Outline 1.1

1. A cost-benefit analysis also makes the exclusionary rule inapplicable to civil deportation proceedings. The reduced deterrent effect of the rule in these cases makes the benefits of using it low, while the costs remain high. INS v. Lopez-Mendoza

vii. Evidence obtained by private persons NO1. The 4th A exclusionary rule does NOT apply to evidence obtained by private persons UNLESS the

person was acting as an agent for the government. Burdeau v. McDowell a. Courts use the totality of circumstances test to determine whether a private person was

acting as a government agent.2. Example: Fedex found open box w/ powder & called cops. Where a private person conducts a

search then calls the police, police activity which is not a significant expansion of the earlier private search is NOT a separate search requiring a warrant. If they had opened at the request of the gov’t, they need a warrant.

viii. Evidence found pursuant clerical mistake NO1. Traffic stop, computer said guy had warrant, D arrested & cop found pot. Warrant was a clerical

error. D wants evidence suppressed. While the exclusionary rule generally applies to government actors, it does NOT apply to evidence obtained due to clerical errors because this would not further the goal of deterring police misconduct. Arizona v. Evans

a. If cop knows there is an error, this is different. If cops reasonably believe they have a basis to search based upon actions of magistrate.

ix. Evidence Obtained by Foreign Officials NO1. The exclusionary rule does NOT apply to evidence obtained by a foreign official, even if the U.S.

was involved in the foreign search, because the 4A protects only “the people,” meaning those people who are sufficiently connected with our national community as to be considered part of it.

2. U.S. v. Esparza-Mendoza (2003); previously deported alien felon who reenters the U.S. is not covered by the 4th A.

f. Herring v. United States (2009) Issue: If cop reasonably believes there is outstanding warrant, but this belief is wrong b/c of bookkeeping error, must the contraband found during a search incident to that arrest be excluded in a later prosecution? Analysis turns on the culpability of the police and the potential of exclusion to deter wrongful police conduct.

i. If this is an honest mistake, this is no different than Leon. Good faith mistake, happens more often now b/c of ID theft.

10. PROTECTED AREAS AND INTERESTS

a. KATZ – BIG CASE TO KNOW FOR EXAM!!!!b. Katz v. United States (1967); FBI Agents, w/o warrant, bugged the outside of a public telephone booth and recorded

Katz’s end of the telephone conversation in that booth.i. Rule: 4th A governs not only the seizure of tangible items, but extends as well to the recoding of oral

statements overheard w/o any “technical trespass.” Government investigative activity that intrudes upon a justifiable expectation of privacy constitutes a search within the meaning of the 4A.

1. What a person knowingly exposes to the public, even in his own home or office, is not a subject to 4A protection But what he seeks to preserve as private, even in an area accessible to the public, may be constitutionally protected.

a. TEST: Whether the individual has a reasonable expectation of privacy that society is prepared to recognize as reasonable.

2. RULE: The 4A protects people not places, so it’s not the phone booth that protected, but Katz that’s protected.

ii. Plain view doctrine If government agents, while acting lawfully, plainly see something which the agents have a probable cause to believe is incriminating, the agents may search, seize and use it as evidence of a crime even without a search warrant

1. What about lip readers or someone overhears? Would not be able to get the info suppressed if they can see or hear you naturally.

iii. Reasonable Expectation of Privacy In order for the conduct of a government agent to constitute a search and seizure under the 4A, there must be a manifest subjective expectation of privacy in the area of the search or the object seized, and this expectation must be one that society is willing to recognize as legitimate

C. WHAT DOES THE EXPECTATION OF PRIVACY EXTEND TO? PRIVACY, LIBERTY INTERESTS & POSSESSORY INTERESTS

14

Page 15: Crim Pro Outline 1.1

i. 4TH AMENDMENT INTERESTS 1. In addition to privacy interests, the 4A also protects a person’s liberty interests and his

possessory interests in his property. United States v. Placea. A seizure that intrudes upon possessory or liberty interests is subject to the 4A even if no

search takes place. Soldal v. Cook County

ii. GARBAGE 1. Cops can’t go down driveway to take garbage, but they can wait for the garbage men to go get it,

or ask the garbage guy to keep it separate. Once it is on the curb it is abandoned and no longer yours. NO EXPECTATION OF PRIVACY IN GARBAGE.

a. The rule applies even if a person takes steps (i.e., fine shredding) to preserve his privacy in his articles before leaving them for garbage pick-up.

2. Having deposited garbage in an area particularly suited for public inspection and public consumption, for the express purpose of having strangers take it, respondents could have no reasonable expectation of privacy in the items that they discarded. California v. Greenwood

III. TELEPHONE #’S

1. There is no expectation of privacy with #’s because it is common knowledge that telephone companies will have that information. Smith v. Maryland

IV. SURVEILLANCE PLANE

1. Plane traveling at 1,000 ft. no reasonable expectation of privacy “because any public flying in this airspace who glanced down could have seen everything that the officers observed.

a. OK even if there is a no trespassing sign. Cops can go in or fly over open fields.

v. CURTILAGE V. OPEN FIELDS 1. Open Fields are not protected by the 4A because society has no interest in protecting privacy

in the types of activities that occur in open fields.a. Curtilage (land immediately surrounding the house) is protected, not the neighboring

open fields. 2. No “search” within the meaning of the 4A where police observe areas within the curtilage from a

vantage point (i.e., hovering in a helicopter at 400 feet) legally available to any member of the public, and where there is no showing that such a vantage point is sufficiently rare to make an expectation of privacy in the area reasonable. Florida v. Riley

vi. BUSINESS AND COMMERCIAL SHOPS 1. Businessmen, like the occupant of a residence, have a constitutional right to go about business

free from unreasonable official entries upon private commercial property.

VII. PRIVATE AREAS IN PUBLIC PLACES

1. There is an expectation of privacy in these places, like public bathroom stalls.

VIII. PRISON CELLS

1. The 4A does NOT protect these areas because security interests trump a prisoner’s privacy interest, and loss of privacy is an inherent incident of confinement.

IX. VEHICLES 1. Exterior No expectation of privacy where police examine the tires and take paint scrapings

from the exterior of a car in a public parking lot. 2. Interior The inside of an automobile, however, IS subject to 4A protection.

x. EFFECTS 1. The 4A DOES protect effects such as personal luggage.

a. While travelers expect that others may move luggage they have placed in an overhead bin, they do not expect them to handle it in an exploratory manner, and officers who do so therefore conduct a 4A search.

xi. ENHANCING THE SENSES: 1. It is not a search for an officer, lawfully present at a certain place, to detect something by one of

his natural senses or common sense-enhancing devices, such as flashlights or binoculars.

2. Kyllo v. United States (2001); W/o warrant, federal agents used a thermal imaging device to determine whether Kyllo’s home was emitting enough relative heat to indicate a marijuana growing operation. Cops then got warrant to search house. In the home, the Court has held that all details are intimate details; b/c the entire area is held safe from prying eyes of the gov’t.

a. KYLLO PROTECTS PLACES. KATZ PROTECTS PEOPLE.

15

Page 16: Crim Pro Outline 1.1

b. RULE: The Gov’t conducts a search when it uses sense-enhancing technology that is not in general public use to obtain information about the interior of a home that it could not otherwise obtain without physical intrusion into the home.

3. ISSUE: As technology increases, what becomes a reasonable search? 4. If Katz protects people and not places, suppose there is no way to provide for privacy given

technological advances. Would the gov’t be able to gather info w/o trespassing w/ sensory enhancing technology? Kyllo does NOT talk about the person’s right to privacy, but about the house.

XII. CANINE “SNIFF”1. A sniff, which reveals only the presence or absence of narcotics, is NOT a 4A search because it

is so limited in both its intrusiveness and in the information it reveals. As long as the sniff was accompanied with a reasonable reason, it will not be considered a search under the 4A.

2. Illinois v. Caballes ; Sniff rule reaffirmed and applied to a dog sniff of a car during a traffic stop.

3. Example: What if the cops had a super sniffer that they could aim at you and detect if you were carrying drugs. If you are carrying cocaine in your pocket, do you have an expectation of privacy for your pocket? Have you lost this expectation of privacy given this advanced technology? Do we let the gov’t use these machines? Need PC for search and the machine would create PC for a search.

xiii. ELECTRONIC TRACKING 1. The use of a beeper to track a container from origin is NOT a 4A search where visual

surveillance would suffice. The 4A does NOT prohibit officers from using sense-enhancing technology to perform investigative tracking which they could fully perform without such technology.

a. Can’t keep the beeper on for too long or it’s invasive.b. However, officers do conduct a 4A search when they secretly use a device to obtain

information which they could not otherwise have obtained without conducting the search with the tracking device (using a tracking device to follow the movement of a chemical to manufacture drugs).

c. Maynard and Jones; holding police conducted a search “by tracking D’s movement 24 hours a day for four weeks w/ a GPS devise they had installed on his Jeep w/o a valid warrant.”

XIV. PHOTOGRAPHIC MAGNIFICATION

1. An aerial photograph of a chemical company’s industrial complex was NOT a search. Dow Chemical v. United States

a. Although the use of technology not generally available to the public might constitute a 4A search, the mere fact that technology provides some sense-enhancement does not necessarily mean its use constitutes a search.

xv. Facial Character Recognition: You don’t have a right not to be photographed out on the street, but there would be an issue if they ran facial character recognition through these cameras.

xvi. Passive Alcohol Sensor: Person could not smell or see the alcohol, but the machine could. Unsettled area. 1. Court hopefully will be leery about this and ask “would I want to be treated like this?”

D. PRIVILEGED MATERIALS

i. Andresen v. Maryland (1976) Police seized papers pertaining to a fraudulent land sale from D’s law office. D challenged the seizure as a 5A self-incrimination violation. In a criminal defense office they would likely find evidence of a crime, so cops are going to want to look at it all – cops will say they are looking for evidence of ‘future crime,’ which is not privileged.

1. RULE: The 5A does not prohibit government seizure of preexisting documents and other tangible items tending to implicate the owner of those documents and items.

a. Supreme Court finds that this is not a 5A violation because he is not being compelled to produce the documents the police are taking the documents. “A party is privileged from producing the evidence but not from its production.”

2. Some states have instituted a “special master” to review documents in an atty’s office.

II. NEWSPAPER REPORTERS—SPECIAL RIGHTS?1. Zurcher v. Stanford Daily ; cops w/ warrant searched newspaper office for film relevant to ID

those how had injured cops at demonstration. Stanford said subpoena us and we will bring it in and the S.C. said it is not the call of the paper; it is the call of the Police or Prosecutor w/ PC. Don’t go into news media often b/c they would be reported in the media.

16

Page 17: Crim Pro Outline 1.1

a. RULE: Newspapers have no 1st A protection.

11. PROBABLE CAUSE

a. PC for an Arrest: For arrest, there must be a substantial probability that a crime has been committed and that the person to be arrested committed it.

b. PC for a Search: For search, there must be a substantial probability that certain items are the fruits, instrumentalities or evidence of crime and that these items are presently to be found at a certain place.

i. Need FACTS to establish PC for a warrant. Need someone who can be checked out. C. WHAT ABOUT AN ANONYMOUS SOURCE?

i. Spinelli Two Prong TEST: 1. If you are going to use an unnamed informant, you have to tell us why you trust the informant (Reliability), and 2. Need sufficient information/facts as to why the informant believes what they are saying (Information).

1. TODAY, must be able to vouch for the informant’s reliability. If it’s a single transaction w/ a single informant, Defense must DEMAND the ID of the informant

d. Spinelli v. United States (1969); Based on a sparsely corroborated tip from an informant, the FBI obtained a search warrant that led to the discovery of evidence tending to indicate that Spinelli was running an illegal bookmaking operation. The informant was unknown and supposedly reliable. Underlying problem with this case is that there is probably enough information to show that there was probable cause that D was booking bets. However, there is no information about the informant – don’t know who he is or how he knows this information. If he had added that “I placed a bet w/ Spinelli this morning,” the affidavit would have been fine.

i. Spinelli Two Prong TEST: test governs search warrants based on informant tips and tells when probable cause has been met: (This test gets to sufficiency of information to reach Probable cause)

1. First, the application must set forth underlying circumstances necessary to enable the magistrate independently to judge the validity of the informant’s conclusion that the information is as he says it is, and,

2. Second, the affidavit must attempt to support the claim that the informant is credible and the information is reliable. Must tell us something about the informant – and they did not have it here.

ii. Thus, the affidavit must state that the informant was reliable (“I placed a bet w/ Spinelli”), or show underlying information that the informant has been helpful and reliable in the past.

e. Illinois v. Gates (1983); Based on a corroborated informant’s tip regarding Gates’ travel plans, police obtained a search warrant and discovered about 350 lbs. of marijuana in Gates’ automobile, which the trial court excluded as fruits of an illegal search. Court did not follow the Spinelli two prong test b/c there is NOTHING to identify to the magistrate that this person is believable. Here, b/c the detailed provided for the other prong is so strong and so specific, that any fair minded magistrate would say “go search.”

i. Exception to Two Prong Rule: One prong can lift up the other prong. It is based on the Totality of the Circumstances.

1. A search warrant based on an informant’s tip may be properly issued if, given the TOTALITY OF CIRCUMSTANCES set forth in the warrant application, including the veracity and basis of knowledge of the informant and any corroboration of the informant’s information, there is a FAIR PROBABILITY that contraband or evidence will be found in the place to be searched. An affidavit must provide the magistrate with a substantial basis for determining the existence of probable cause; his action cannot be a mere ratification of the bare conclusions of others.

2. Magistrate issuing warrant;a. A magistrate’s determination of probable cause should be paid great deference by

reviewing courts,b. TEST: Did the magistrate make a practical, common-sense decision, whether, given all

the circumstances set forth in the affidavit before him, including the veracity and basis of knowledge of persons supplying hearsay information, there is a fair probability that evidence of a crime will be found in a particular location.

- Provides a certain degree of discretion to the judge- A judge can ask questions in an attempt to fill in gaps in order to reach

probable cause. The caveat is that the information must be under oath. - One of the issues that the criminal defense community is skeptical

about is whether an actual AI exists. Maryland v. Pringle – the holding of this case allows for innocent people to be found guilty by association.

F. INFORMANT’S INFORMATION

17

Page 18: Crim Pro Outline 1.1

i. Massachusetts v Upton ; (1984) after searching motel room w/ valid warrant and finding stolen goods, cops got call from D’s ex-girlfriend w/ info about location of remainder of stolen items. S.C. upheld the warrant to search motor home based on ex’s info b/c there was a “fair probability” that contraband or evidence of crime would be found in motor home.

ii. A search warrant may become invalid if the information on which it rests becomes “stale,” so that there is no longer probable cause to believe the evidence is in the place stated in the warrant. Schmidt v. State

G. PROBABLE CAUSE STANDARD FOR WARRANTS

i. U.S. v. Grubbs (2003); Warrants require the magistrate to determine (1) that it is now probable that (2) contraband, evidence of a crime, or a fugitive will be on the described premises (3) when the warrant is executed.

H. FALSE AFFIDAVIT SITUATIONS

i. Franks v. Delaware ; If a challenger of a warrant application proves by a preponderance of the evidence that the affiant made a false statement knowingly and intentionally, or with reckless disregard for the truth, and that absent that statement, the application does not establish probable cause, then the court must void the search warrant and exclude the fruits of the search.

1. RULE: If the officer knowingly made false statements on the affidavit, then the warrant may result in the suppression of any evidence recovered via the warrant.

2. BLUTH: Affidavit says informant bought drugs 24 hours ago from P. What if the information in the search warrant is not correct in regards to “P did not sell drugs to anyone 24 hours before, P was in Florida when informant said he bought drugs?” Leon says if the cops act in good faith there is no reason to suppress. The other argument is if the cops lied to the judge they should be accountable.

a. Issue: Did cops know informant was lying? What are circumstances around informant?b. If you can make a clear showing that the information in the affidavit is false, you are

entitled to a hearing and entitled to have the informant produced. If information is stricken from affidavit, what is left?

i. INFORMANT’S PRIVILEGE i. Defendants do not have a right to confront the “reliable” informant whose tip the police relied on in

their warrant application. McCray v. Illinois1. The judge who hears the motion to suppress may decide whether he needs the informer’s identity

to decide whether the officer is a credible witness.ii. Some courts encourage suppression hearing judges to make an in camera inquiry of the confidential

informant, without the defense present, to determine the truth of an officer’s prior assertions regarding that informant. People v. Darden

iii. Maryland v. Pringle (2003); Car stopped for speeding. When driver opened glove box to get registration, cop saw big roll of cash. No outstanding warrants, but driver consented when cop asked to search car. Cop found $ and cocaine in back seat armrest. 3 men all denied drugs were theirs, cops suspected “common enterprise”. After arrest, passenger Pringle (front seat passenger), gave oral confession that drugs were his – he then tried to have confession suppressed claiming it was the fruit of an illegal arrest b/c no PC to arrest Pringle.

1. PC Standard: The substance of all the definitions of PC is a reasonable ground for belief or guilt.” and that the belief of guilt must be particularized w/ respect to the person to be searched or seized.

2. Outcome: It was an entirely reasonable inference from the fact that any or all 3 men had knowledge or, and exercised dominion and control over, the drugs. Reasonable cop could conclude there was PC to arrest P.

3. Exception: Any inference that everyone on the scene of a crime is a party to it must disappear if the Gov’t informer singles out the guilty person (or the cops know who is guilty).

a. Ybarra ; NO GUILT BY ASSOCIATION. Where the standard is PC, a search or seizure of a person must be supported by PC particularized to that person.

b. Di Re ; about counterfeit gas coupons, gov’t knew who was guilty but arrested all passengers.

J. OTHER SOURCES OF PROBABLE CAUSE

I. ALLEGED VICTIM OF, OR WITNESS TO, A CRIME

1. An ordinary citizen may report a crime as a victim or witness. Such a citizen is presumptively reliable; no showing of prior reliability is necessary. State v. Paszek

18

Page 19: Crim Pro Outline 1.1

2. The issue in most victim/witness cases is whether the arrest was justifiable based on the description the victim or witness gave. Probable cause to arrest may exist despite some discrepancies between the description and the suspect. Brown v. United States

II. DIRECT OBSERVATIONS BY THE POLICE 1. Guy carries record player w/ tags. An officer’s own observations may even supply probable cause

& running amounts to PC. In such a case, the probable cause determination must rest on the standards of a reasonable, cautious and prudent officer, and not those of a casual passerby. Brooks

III. ORDER FROM OFFICIAL CHANNELS

1. Officers who make an arrest based on information or orders from officials, such as police bulletin, may assume PC existed to support the orders. However, if the arrest turns out to be illegal b/c PC did not exist, the officers’ entitlement to rely on official channels will not turn the illegal arrest into a legal one. Whitely v. Warden

K. ISSUANCE OF SEARCH WARRANTS

i. Neutral and Detached Magistrate1. Must be someone who is not being paid by the warrant. 2. U.S. v. Scott ; (2001); A “special judge” has authority to issue a warrant ONLY when the regular

general sessions judge was unavailable. 3. The Constitution permits only a neutral and detached magistrate to issue search warrants. Failure

to comply with this will result in EXCLUSION no matter how strong probable cause. Coolidge4. Bluth says must be a judge for a search warrant. On the other hand, for an arrest warrant, the

issuing official need not be a judge or even a lawyer, but must only be (1) neutral and detached, and (2) capable of determining whether probable cause exists for the requested arrest of search.

a. Can’t be someone who is getting paid to issue a warrant. 5. A warrant issued by an official who fails to make any sort of probable cause determination (didn’t

read affidavit), but acts simply as a ‘rubber stamp,’ is INVALID even if probable cause did exist. Rooker v. Commonwealth

ii. Description of Place to be Searched1. The 4A requires that search warrants describe with sufficient clarity the place to be searched

and the thing to be seized. The description is sufficiently particular if the officer can, with reasonable effort, ascertain and identify the place intended.

2. The validity of the warrant must be assessed on the basis of information that the officers disclosed, or had a duty to discover and to disclose, to the issuing magistrate. An officer’s failure to realize the Overbreadth of a search warrant will not make the search invalid if such failure is objectively understandable and reasonable. Maryland v. Garrison

iii. Particular Description of Things to be Seized 1. The 4A also requires that search warrants describe with particularity the things to be seized .

a. The degree of particularity necessary varies with the type of items to be seized. They can only look in places where the particular item could be found.

b. Can’t look for Corvette engine in drawer. If they are looking for policy slips, open up bedroom drawer and found cocaine, they can take the cocaine.

c. RULE: If they are searching in a place authorized for them to look pursuant to the warrant, anything they find in plain view that is clearly contraband or evidence of a crime can be seized or used as evidence in a prosecution.

2. Rationale: To prevent general searches, to prevent the seizure of objects on the mistaken assumption that they fall w/in the magistrate’s authorization, and to prevent the issuance of warrants on loose, vague or doubtful bases of facts.

iv. Particular Description, Reliance on Affidavit1. Groh v. Ramirez ; Issue was whether a defective warrant can be overcome by a sufficient

description in the supporting affidavit. Warrant MUST describe the items to be seized.

l. TIME OF EXECUTION i. Search warrant usually must be executed within a certain time and that time is proper, “provided that the

probable cause recited in the affidavit continues until the time of execution, giving consideration to the intervening knowledge of the officers and the passage of time.” United States v. Nepstead

- Generally search warrants must be executed between 10 days.ii. Contraband

1. Particularly in these cases, the issue of “staleness” becomes an issue.iii. Day vs. Night

19

Page 20: Crim Pro Outline 1.1

1. Generally, daytime entry is preferred. Reason is to protect the cops.2. Permission from the judge is required , which may require a heightened probable cause standard,

to search at night because of the possible dangerous ramifications of entering someone’s house at night.

iv. “Sneak and Peak”1. These type of warrants, which authorize officers to search when the occupants are absent and

without leaving notice of the search, also involve special requirement There must be a showing of reasonable necessity for the delay in giving notice, and notice must be given within a reasonable time after the search. United States v. Villegas

m. GAINING ENTRY i. The 4A’s reasonableness requirement permits officers to break open a door to execute a warrant, but

requires, in general, that they first “knock and announce” their presence and authority. 1. What must be done if an officer wants to enter someone’s house unannounced?

a. “In order to justify a “no-knock” entry, the police must have a reasonable suspicion that knocking and announcing their presence, under the particular circumstances, would be dangerous or futile, or that it would inhibit the effective investigation of the crime by, for example, allowing the destruction of evidence.

i. This showing is not high, but the police should be required to make it whenever the reasonableness of a no-knock entry is challenged.

2. U.S. v. Banks (2003); (i) How long a wait is necessary b/f cops may reasonably conclude they have been refused admittance? Based on the facts known by the cops, must decide whether the occupant’s failure to admit fairly suggests a refusal to let the cops in. Varies by hotel room v. town home, etc. (ii) What shorter wait will suffice b/c of what kind of pressing circumstances? Depends on the pressing circumstances and whether evidence can be destroyed during wait.

II. SEARCH OF PERSONS ON THE PREMISES

1. A warrant to search premises does NOT authorize a search of the occupants therein absent probable cause to do so. A person’s proximity to criminal suspects does NOT alone give rise to such probable cause. Ybarra v. Illinois

2. Terry v. Ohio ; To pat down a person for weapons there must be a reasonable belief that he was armed and presently dangerous.

3. Mistake Case: L.A. County v. Rettele (2007); Cops got it wrong? Warrant to search 4 black suspects and their homes. Cops find white man/woman in one of the houses and have them stand naked for 2 minutes (they just bought the house). Supreme Court said this was OK to protect the safety of the cops and pple.

iii. DETENTION OF PERSONS ON THE PREMISES 1. HOWEVER, police may detain occupants of a dwelling while they execute a search warrant there. 2. Michigan v. Summers ; Holding a person during the duration of the search is less invasive then

searching the house. 3. Muehler v. Mena ; Ms. Mena and 3 others were handcuffed for 2-3 hours during search. Court

found such action was reasonable b/c the gov’tal interests outweighed the marginal intrusion,” for in “such inherently dangerous situations, the use of handcuffs minimizes the risk of harm to both cops and occupants” and was all the more reasonable b/c of the need to detain multiple pple.

iv. INTENSITY AND DURATION OF THE SEARCH 1. Everything may not be looked at while searching the premises, unless,

a. The warrant specifically allows this, ORb. The items are in plain view while performing the search.

2. Once the items named in the search warrant have been found, the search must cease.

v. SEIZURE OF ITEMS NOT NAMED IN THE SEARCH WARRANT 1. Where officers may search depends on the items listed in the warrant. Officers may only search in

places in which the items may be concealed.a. The 4A does NOT include an inadvertent discovery requirement for seizing items that are

not listed in a search warrant. Horton v. Californiab. As long as officers confine their search, in area and duration, to the terms of the warrant

or a valid exception to the warrant requirement, they may seize contraband or evidence they discover which is not listed in the warrant whether they suspected they would discover the items or not.

vi. PRESENCE OF 3D PARTIES

20

Exception to the “knock-and-announce” rule

Page 21: Crim Pro Outline 1.1

1. 3d parties may be present during officers’ execution of a warrant, such as for purposes of identifying stolen property.

2. However, the 4A requires their presence to be in aid of the officers’ execution.a. Presence of the media during execution of a warrant, not being such an aid, is thus

unconstitutional. Wilson v. Layne

vii. DELIVERY OF WARRANT 1. Groh v. Ramirez ; A search warrant must be delivered at the place searched.

viii. COMPUTER SEARCHES 1. State v. Evers (2003); Search warrant affidavit indicated AOL provided the billing address for e-mail

used in child porn site.

2. U.S. v. Riccardi ; Warrant listed items to be seized as the target computer “ and all electronic and magnetic media stored therein, together w/ all storage devises, internal or external to the computer.

n. PREFERENCE FOR WARRANTS i. The Supreme Court has expressed a strong preference for searches made pursuant to a warrant—although

there are so many exceptions to the warrant requirement that police make most searches and seizures without one.

12. WARRANTLESS ARRESTS AND SEARCHES OF THE PERSON

I. PROBABLE CAUSE 1. United States v. Watson (1976); A federal postal inspector, acting on a tip from a reliable informant, arrested

Watson without a warrant even though the inspector had time to secure a warrant. Watson challenged the warrantless search on constitutional grounds. There was no good reason for the cop to not have a warrant.

a. RULE: An officer with probable cause to make a FELONY arrest is NOT required, under the 4A, to obtain an arrest warrant (in a public place).

2. MISDEMEANOR CASES

a. Mom driving w/ no seat belt. The 4A also permits warrantless arrests for misdemeanor, including misdemeanors that do not involve breaches of the peace. Atwater v. City of Lago Vista

i. For misdemeanors, some state courts require that the offense be committed in the officer’s presence before he can make a warrantless arrest, while others have held that the 4A does not include an “in presence” requirement.

ii. Being arrested for a misdemeanor really matter b/c you will be searched! You can get arrested if a misdemeanor is committed in the cops presence.

3. FLEEING FELON AND DEADLY FORCE

a. The use of deadly force to prevent the escape of all felony suspects, whatever the circumstances, is constitutionally unreasonable. However, where the officer has probable cause to believe that the suspect poses a threat of serious physical harm, either to the officer or to others, it is not constitutionally unreasonable to prevent escape by using deadly force.

b. The 4A reasonableness standard applies to all claims of excessive force during an arrest or other seizure of a person, and asks whether the officer’s actions are objectively reasonable in light of all the facts and circumstances. Graham v. O’Connor

c. Scott v. Harris ; (2007) Scott (cop) rammed and injured Harris, who was recklessly speeding and placing others lives in danger. B/c Harris posed an actual and imminent threat to the lives of other citizens and the cops, it was reasonable for Scott to take the action he did.

4. JUDICIAL DETERMINATION

a. Gerstein v. Pugh ; A policeman’s on-the-scene assessment of probable cause provides legal justification for arresting a person suspected of crime, and for a brief period of detention to take the administrative steps incident to arrest.

i. Rule: The 4A requires a judicial determination of probable cause as a prerequisite to extended restraint on liberty following arrest.

ii. How long can the person be detained for? 48 hours is cool.

ii. SEARCH INCIDENT TO LAWFUL ARREST

1. United States v. Robinson (1973); (Like the Paris Hilton arrest where she was found w/ cocaine in her purse). Officer Jenks saw Robinson driving at 11p.m. Four days earlier, Jenks had checked Robinson’s drivers license and he knew it was revoked (an offense w/ min. sentence). Jenks pulled Robinson over and during a pat-down incident to Robinson’s arrest for driving w/o license, Jenks removed a crumpled cigarette pack from Robinson’s

21

Page 22: Crim Pro Outline 1.1

pocket, opened it, and found heroin. A police officer’s determination as to how and where to search the person of a suspect who he has arrested is necessarily an ad hoc judgment which the 4A does not require to be broken down in each instance into an analysis of each step in the search.

a. Rule: Any lawful custodial arrest justifies a full search of the arrestee’s person. It is well settled that a search incident to a lawful arrest is a traditional exception to the warrant requirement of the 4A.

b. Rationale: The act of arrest (taking someone in to jail) is a high invasion of privacy, and thus searching the contents on the arrestee is a far less invasion of privacy then arrested. Need to disarm the suspect in order to take him into custody, need to preserve evidence on his person for later use at trial.

c. Distinct Propositionsi. First, the search may be made OF THE PERSON of the arrestee by virtue of the lawful arrest,

ii. Second, the search may be made of the area WITHIN THE CONTROL of the arrestee. d. DISSENT: There is always the possibility that a cop, lacking PC to obtain a search warrant, will use a

traffic arrest as a pretext to conduct a search.

e. Notes:i. This search incident to arrest rule applies even if the arrest is for an offense that is “trivial,” such

as one carrying no mandatory minimum sentence. Gustafson v. Floridaii. The 4A permits custodial arrest (and all the search and seizure power that goes with it) even if

conviction could not result in jail time and where there is no compelling need for immediate detention. Atwater v. City of Lago Vista

iii. Virginia v. Moore (2008); D arrested for driving on suspended license although state law mandated that a summons be utilized in the circumstances of the case. State court held evidence found in a search incident to that arrest had to be suppressed. Unanimous S.C. concluded that neither the arrest nor the search was unconstitutional.

2. OFFICER MOTIVATION TO MAKE STOP

a. As long as there is PC to stop someone, even if the motivating factor in stopping the person was to search the person, it’s cool. Whren v. United States (1996)

b. Ashcroft v. all-Kidd ; (2011) AG authorized feds to use the federal material witness law to detain terrorist suspects. S.C. found this case involved a valid warrant based on individualized suspicion & was OK.

3. OTHER ISSUES REGARDING SEARCHES OF AN ARRESTEE

a. Effects in an arrestee’s possession that are subject to search and seizure at the time and place of arrest remain subject to search and seizure without a warrant at the stationhouse, or even after incarceration, even if a substantial period of time has elapsed between the arrest and the search and seizure. United States v. Edwards

i. Bluth says they can search the effects for anything. However, a search of an arrestee’s property for evidence of a separate crime requires probable cause. People v. Trudeau

b. Where PC justifies a blood test for alcohol content, emergency makes the test permissible because there is a danger of destruction of the evidence if the officer must wait for a warrant.

i. However, courts must evaluate the reasonableness of surgical intrusions as searches on an ad hoc basis. Winston v. Lee

ii. Because such substantial intrusion as surgery to remove a bullet suspected to be evidence of a robbery requires a compelling need for it, it is unreasonable if other evidence is sufficient. Winston v. Lee

c. An officer cannot justify a search as incident to arrest after he decides to issue a citation in lieu of making an arrest. Knowles v. Iowa

i. Where there is probable cause to arrest a person, but he voluntarily appears and is not under arrest, a brief, limited search of his person (i.e., fingernail scrapings) to preserve highly perishable evidence is permissible without a warrant. Cupp v. Murphy

13. WARRANTLESS ENTRIES AND SEARCHES OF PREMISES

a. Chimel v. California (1969); Man was arrested, with an arrest warrant, at his home and the police then proceeded to search his entire house without a search warrant. In this case, the police did not have the justification to search the arrestee’s entire house.

i. “Immediate Control” Test: The search will only be justified when there is a search of the arrestee’s person and the area “within his immediate control”—construing that phrase to mean the area from within which he might gain possession of a weapon or destructible evidence.

b. Arizona v. Hicks , No such thing as a crime scene exception. They can’t go back w/o a warrant and gather evidence.

I. SEARCH OF PREMISES INCIDENT TO AND AFTER ARREST

1. “IMMEDIATE CONTROL”

22

Page 23: Crim Pro Outline 1.1

a. The area within an arrestee’s ‘immediate control’ may include a dresser drawer and purse in the room where the arrest took place, even if the search takes place when the arrestee is outside the room in handcuffs.

b. Officers may also search other areas of an arrestee’s house which comes within his immediate control as the arrestee moves to another area to get dressed.

i. D arrested at front door went into bedroom to change into street cloths; HELD, police could look into dresser drawer D was about to open, gun found admissible. D did not have right to say he would go to jail naked.

2. PROTECTIVE SWEEP

a. Officers may conduct a protective sweep of adjoining spaces incident to an arrest.i. Officers may extend the sweep to other areas on reasonable suspicion that another

may be present and pose a danger.ii. However, they must limit such a sweep to spaces that may harbor a person, and may

continue only as long as necessary to dispel the suspicion, and never longer than it takes to complete the arrest and depart.

3. PURSUING AN ARRESTEE a. Cop sees college kid w/ Gin. Enters dorm and sees pot plant. An officer may enter premise

incident to an arrest elsewhere to monitor the movements of the arrestee to prevent escape, injury, or destruction of evidence any evidence the officer discovers in plain view or as a result of a Chimel search while monitoring the arrestee’s movements is admissible. Washington v. Chrisman

II. WARRANTLESS SEIZURES WHILE IN PREMISES TO ARREST

1. Kentucky v. King ; (2011) Cops saw X buy crack and radioed for backup. B/u lost sight of X, heard door at end or corridor slam. Two doors, cops bust down door w/ pot smoke coming out the bottom and arrest King. X was behind other door. Kentucky court suppressed evidence and reversed based upon the “Police-created exigency” doctrine: Police may not rely on the need to prevent destruction of evidence when that exigency was “created” or “manufactured” by the conduct of the police.

a. Rule: The exigent circumstances rule applies when the cops do not gain entry to premises by means of an actual or threatened violation of the 4th A.

b. Analysis: Here, King chose not to stand on his constitutional rights but instead elected to attempt to destroy evidence; he has only himself to blame.

III. WARRANTLESS SEARCHES OF PREMISES UNDER CERTAIN CIRCUMSTANCES

1. STREET ARREST: An arrest on the street does not itself provide an exigent circumstance justifying a warrantless search of the arrestee’s house. Vale v. Louisiana

2. DESTRUCTION OR REMOVAL OF EVIDENCE a. Where officers reasonably fear the destruction or removal of evidence, they may secure

the premises, from inside or outside, while they AWAIT a search warrant. (This is what the Vale cops should have done!) United States v. Grummel; Segura v. United States

b. Securing Premises without Arrest. Illinois v. MacArthur A man’s wife exits her house and informs police that her husband has drugs hidden in the house.

i. Even if there is no arrest, the 4A permits officers to secure premises while awaiting a search warrant where:

1. They have probable cause to believe the premises contain evidence of a crime or contraband;

2. They have reason to fear the destruction of evidence if they leave to get a warrant;

3. They make reasonable efforts to limit the intrusion upon privacy; AND4. They secure the premises only for as long as reasonably necessary to

obtain the warrant

3. SERIOUSNESS OF OFFENSE NOT DETERMINATIVE

a. The seriousness of the offense (i.e., homicide) does not alone create exigency justifying a warrantless search. OK to seize evidence in plain view or w/in the crime scene area. Mincey v. Arizona

b. Brigham City v. Stuart ; (2006) Cops may enter a home w/o a warrant to render emergency assistance to an injured occupant or to protect an occupant from imminent injury.

IV. ARREST WARRANT NEEDED TO ENTER PREMISES

23

Page 24: Crim Pro Outline 1.1

1. Payton v. New York (1980); This involves companion cases in which police entered homes to make an arrest without arrest warrants, found evidence of crimes therein and used that evidence to convict the Ds. Cops were lazy in not getting warrant. Watson said you don’t need an arrest warrant in PUBLIC to arrest someone.

a. General Rule absent special circumstances, a warrantless entry to search for weapons or contraband is unconstitutional even when a felony has been committed and there is probable cause to believe that incriminating evidence will be found within.

i. What is lost if the arrest is unlawful?1. Any evidence obtained, whether in plain view or not, and regardless

whether it was in the person’s immediate control,2. Any search of the person incident to the unlawful arrest,3. Any statements made contemporaneous to the unlawful arrest is out.4. HOWEVER the prosecution may still continue the proceedings

against the person, but the evidence which has resulted from the unlawful arrest will be excluded.

2. EXCEPTIONS:a. HOT PURSUIT: When officers are in hot pursuit and delaying entry of a dwelling could

endanger the lives of the officers or others, a warrantless entry is permitted. Warden v. Hayden

b. DOORWAY OF HOUSE: Police may also make a warrantless arrest of a person who is standing in the doorway of her house, exposed to the public as if she were outside. United States v. Santana

i. If she attempts to go into the home when the police approach, Hayden’s hot pursuit rule permits them to follow her in to make the arrest.

c. SERIOUSNESS OF OFFENSE?: The seriousness of the offense is an important factor when determining whether the special circumstances exception applies. Welsh v. Wisconsin

i. This exception RARELY applies to justify a warrantless home entry when there is only probable cause for a minor offense.

d. ENTRY INTO 3D PARTY’S DWELLING: Absent special circumstances, an arrest warrant does not authorize entry into a 3d party’s home, even on a reasonable belief that the suspect is present. Steagald v. United States

i. Police must obtain a search warrant to search the premises for the suspect.

14. WARRANTLESS SEIZURES AND SEARCHES OF VEHICLES AND CONTAINERS

a. California v. Carney (1985); Police with probable cause to believe Carney was dealing drugs searched his motor home without a warrant and discovered illegal narcotics therein.

i. Automobile Exception to Search Warrant Rule: Any vehicle that is readily mobile and subject to the pervasive laws regulating motor vehicles may be searched, without first obtaining a search warrant, so long as there is probable cause supporting the search. Mobility is the key factor.

ii. Rationale: The Court has recognized that privacy interests in an automobile are protected; HOWEVER, it the ready mobility of the automobile justifies a lesser degree of protection of those interests b/c of the capacity to be “quickly moved.”

1. This lesser expectation has been applied to:a. Content of a locked car trunk,b. A sealed package in a car trunk,c. A closed compartment under the dashboard,d. The interior of a vehicle’s upholstery, ande. Sealed packages inside a covered pickup truck.

iii. In this case, the court holds that a MOBILE HOME falls under the automobile exception. THUS, the search was not unreasonable

B. NOTES AND APPLICATION OF CARNEY i. The automobile exception does not require a separate finding of extreme circumstances. Dyson

ii. This exception also applies to the warrantless seizure of the automobile itself on probable cause that it (the car) is contraband. Florida v. White

iii. A warrantless search of an automobile must be limited in scope to places in which the items for which probable cause exists might be found. Maldanado v. State

C. SEARCH OF CLOSED CONTAINERS IN CARS i. California v. Acevedo (1991) Cops had PC to search a bag in the trunk of Acevedo’s car for drugs, stopped

the car, opened the trunk and the bag, and found drugs. Problem per Bluth , if they can’t stop him on the

24

Page 25: Crim Pro Outline 1.1

street to search the package, why can they search the package once he puts it in the car. If the cops have PC that drugs are in the briefcase on the street, they can arrest you; if there are drugs you are caught, if there are not drugs it was a bad arrest and you can sue for damages.

1. Rule: Police may search a closed container in a car without a warrant if they have PC to search the container.

2. Rule: if car is pulled over pursuant to traffic stop cops can see what is in plain sight, but can’t search w/o PC.

3. Rule: If on the other hand, if the cops have probable cause to believe there is contraband in a car they can search the car w/o a warrant – b/c car is movable.

4. Problem per Bluth, if they can’t stop him on the street to search the package, why can they search the package once he puts it in the car.

Probable Cause = More Likely Than Not, judged on what the police knew prior to the search.ii. Does Acevedo apply to every sort of container?

1. U.S. v. Burgess ; Cops searched motor home b/c of PC of drug sales, took computer and hard-drives and found child porn. Gov’t said b/c this is automobile cops can search computer w/ PC.

iii. Distinction between packages within car unnecessary:1. Cops searched passengers purse left in car – found drugs. RULE: when there is PC to search for

contraband in an car, Cops may search containers WITHOUT showing of individualized PC for each one, and regardless of whether the container belongs to the driver, a passenger, or someone else. Wyoming v. Houghton

iv. Plain view in car:1. Search of balloon suspected of containing heroin. Held: warrantless searches of containers may be

justifiable under the PLAIN VIEW doctrine, or b/c the contents of the container can be inferred from its outward appearance and location. Texas v. Brown

D. SEARCH OF CAR AFTER ARREST OF OCCUPANT

i. Arizona v. Gant ; (2009) Gant arrested for driving w/o license and placed in back of patrol car. Cops then searched his car and found cocaine in pocket of jacket on the backseat.

1. Rule: Cops may search a car incident to a recent occupant’s arrest (here arrested for driving w/o license, so what were they searching for? Whren said you could search the D when arrested, but not the car) only if the arrestee is w/in reaching distance of the passenger compartment at the time of the search or it is reasonable to believe the car contains evidence of the offense of arrest or a weapon. When these justifications are absent, a search of an arrestee’s car will be unreasonable unless the cops obtain a warrant.

E. SEARCH OF CAR AFTER IMPOUNDMENT

i. Colorado v. Bertine (1987); Bertine was pulled over and arrested for DUI; before the tow truck arrived to take the truck, the officer inventoried the contents of the truck and found drugs.

1. Rule: An inventory of a vehicle and the containers therein PURSUANT TO ESTABLISHED police department administrative POLICY does NOT violate the 4A’s prohibition of unreasonable searches and seizures.

2. Rule per Bluth: No Policy, No Inventory! The inventory search must be a routine, necessary practice. Can’t be done selectively based on demographic. Inventory OK as long as it’s designed to protect the police from claims of theft, and how can you protect from claims of theft if you don’t search all cars. Remember, if you are arrested for a crime that does not have physical evidence (DUI), why would they search.

3. Rationale: This is to protect the police department from false claims of theft.

ii. Dyke v. Taylor Implement Mfg. Impoundment of the vehicle must be justified, or the search of the vehicle will be invalid.

iii. Ex Parte Boyd The inventory search must be done contemporaneously or soon thereafter the impoundment or it will be invalid. Can’t have delay b/c would negate the rationale of search.

F. STOP AND FRISK

i. Terry v. Ohio (1968); 3 men were standing outside engaging in suspicious activity where an on-looking police officer thought they were ‘casing’ the store. The police officer approached the men and frisked Terry finding a pistol. Terry was convicted of carrying a concealed weapon. 4th A not violated.

1. Issue: Whether Terry’s right to personal security was violated by an unreasonable search and seizure.

2. Problem per Bluth: Departs from 4th A PC standard.

25

Page 26: Crim Pro Outline 1.1

3. New Standard: Having reasonable suspicion (based on articulable facts rather than a mere hunch [TOTALITY OF CIRCUMSTANCES]) that the suspect is planning or carrying out illegal activity, the officer may detain to conduct an investigation. Once detained, if the cops think D is armed, cop can pat down outer clothing to see if he has a weapon.

a. If there is reasonable suspicion, the police can detain you for a reasonable amount of time to investigate (detention means that the person cannot walk away – have to wait).

b. There must be a narrowly drawn authority to permit a reasonable search for weapons for the protection of the police officer, where he has reason to believe that he is dealing with an armed and dangerous individual, regardless of whether he has PC to arrest for a crime.

4. Need to answer 2 questions: is there reasonable suspicion (objective standard), and has there been a detainment.

a. Not detained if you are free to leave. i. Example: If police walk up to you on the street and ask you if you have drugs

in your bag, then you are free to leave.

5. What can officers do under Terry?a. Pat down if he has reason suspicion to believe that the person is armed.b. Patting down is limited to the outer clothing of the person – cannot dig through the

pockets.i. If Cops feel a weapon, then they can go into the pocket and get it (if feel a

package that feels like drugs, cannot get UNLESS they know that it is drugs hard to prove).

c. May come into effect when there has been a crime and the victim gives a description of the criminal, and the police find someone matching in the vicinity. The police can hold that person until the victim can come over and identify whether or not he is the perpetrator. If the victim says yes, then there is probable cause for arrest.

6. White Concurrence: Refusal to answer the police is not a basis for arrest. This is changed by the case in the supplement.

G. SIGNIFICANCE OF STOP AND FRISK CASES

1. The Utility of the Balancing Test: The balancing test weighs the suspect’s interest in being free to carry on his business VS. the State’s interest in effective law enforcement in light of the intrusiveness of the stop and the circumstances surrounding it.

2. POLICE ACTION SHORT OF SEIZURE a. Cops on Bus. Seizure of a person does not occur when an officer merely approaches an individual and

begins to ask questions As long as a REASONABLE person would feel free to disregard the officer and go about his business, the encounter is consensual and no reasonable suspicion is required. Bostick

b. A seizure does not occur until the MOMENT officers actually stop a suspect by physical force or the suspect submits to their assertion of authority Police pursuit does not amount to a seizure So a fleeing person who throws drugs while running has not been seized and the 4A will not apply to the evidence. California v. Hodari

C. COMPARE THE SITUATIONS:i. Cops see a guy walking down the street carrying a stereo w/ all the wires hanging out, even

though it looks like it might be stolen, the officer can only ask questions, but cannot detain b/c there is no reasonable suspicion that the person is armed or did anything wrong.

ii. BUT – if the same guy drops the stereo and starts running as soon as they see the TV, then there is reasonable suspicion to believe that there is something going on.

3. GROUNDS FOR TEMPORARY SEIZURE FOR INVESTIGATION

a. Generally the grounds necessary to justify a temporary stop for investigation is less than that required for a full arrest. United States v. Cortez

i. For a Terry-stop, the totality of the circumstances, as understood by a trained law enforcement officer, must raise a suspicion, on a particularized basis, that the individual being stopped is engaged in criminal activity.

b. Cop who sees a person talking with drug addicts, but does not hear the conversation or see anything pass between them, does NOT have grounds for a stop. Cops could only stop and ask him, but reasonable suspicion did not give cops the right to search. Sibron v. New York

c. An anonymous tip is NOT sufficient to give an officer the authority to make an initial stop unless it contains some indicia of the reliability of the tipster, such as the prediction of future activity corroborated by police surveillance information about a person’s current location and appearance are NOT sufficient indicia of reliability to justify a stop in most cases. Florida v. J.L

26

Page 27: Crim Pro Outline 1.1

d. Virginia v. Harris (2009); S.C. denied cert. of VA case which held an anonymous tip of drunk driving insufficient for cops to stop suspect.

e. Among the considerations that are relevant in determining whether there is reasonable suspicion for a stop are presence in a high crime area and evasive behavior. Unlike a mere refusal to cooperate, unprovoked flight from police, whenever it occurs, suggests wrongdoing. Illinois v. Wardlow

f. Palacios v. Burge (2009); Challenged seizure did not violate 4th A simply b/c it was made w/o individualized suspicion where stabbing suspects and cops knew men were in club, and all men in the club were lined up for ID. Contemporaneous to stabbing, cops knew Hispanic suspects were w/in club, witnesses/Vs would not be available to ID for long.

g. Derischsweiler v. State (2011); OK to stop D and ask D what he is up to, when he had been observed stopping next to cars in parking lots and staring at the occupants.

h. Yet compare to ..re Herman (1974); Not OK to stop 15-yr old Ds they all changed directions to avoid the cops. Must be suspicion of a particular crime.

4. PERMISSIBLE EXTENT OF TEMPORARY SEIZURE

a. Generally: An investigative stop must be temporary and last no longer than is necessary to effectuate the purpose of the stop, and the officers must use the least intrusive investigative methods reasonably available to verify or dispel their suspicion in a short period of time. Florida v. Royer

b. There is NO rigid time limitation for a Terry-stop:i. BUT…20 minutes is about the max for a detention, otherwise it becomes an arrest.

ii. If you are stopped for traffic ticket, and then 30 minutes later the drug dog comes to sniff, you can only be held for no longer than is necessary to effectuate the purpose of the stop – so here the search is NO GOOD. Cops must be able to get the dog to your car w/in the time it normally takes to write a speeding ticket. This is allowed b/c there is no additional detention; you are already detained so they can fish during your detention.

c. Modern Problems with Terry:i. What can they do upon a stop for a traffic violation?

1. Can they search your car or ask for permission to search the car?a. Have you been detained? Yes, not free to leave.b. If you are detained can they demand to search if they don’t have a reason to

search?2. Assuming a traffic violation is a Terry detention, how long can they keep you there

and for what purpose? Being litigated more today.

Reasonable Suspicion = Articulable Facts based upon an officers experience and suspicions that leads him to believe crime is afoot.

d. Hiibbel v. Sixth Judicial District Court ; D was convicted of obstructing an officer in his duties for not complying w/ the command in Nevada’s “Stop and Identify” statute that a person lawfully stopped under Terry “shall ID himself.” According to Terry you did not have to identify yourself if the police ask you your name, but under the state statute, it was a separate offense to not identify yourself. The question is whether or not the state can do that?

i. If they can detain you under Terry (because there is REASONABLE SUSPISION to stop you), the state can require you to identify yourself.

1. Compare Callander:a. Black man who runs after he works his night shift runs in nicer

neighborhoods for safety is stopped multiple times by police officers for being out, but he refused to ID himself. He was arrested for failing to ID.

b. Here, there was no reasonable suspicion that the man did anything (where in Hibbel, there was notification to the police that there had been a crime in the area).

e. Illinois v. Caballes ; (2005) D stopped for speeding and drug dog brought to his car to sniff. Held: dog sniff did not change the character of the traffic stop.

f. Ohio v. Robinette ; Pulled over, 45 minutes later the dog comes to sniff. i. RULE: 15 minutes max. This is too long. Remember, the dog sniff is not a search.

5. TEMPORARY SEIZURE OF EFFECTS

a. Arizona v. Johnson (2009); Passenger frisked and gun found. Terry prerequisites for a frisk are (1) a lawful stop, and (2) grounds for a frisk. In a traffic stop, prong 1 is met for all occupants when car

27

Page 28: Crim Pro Outline 1.1

lawfully stopped. To satisfy prong 2, Cops must harbor reasonable suspicion that the person subjected to the frisk is armed and dangerous.

b. Generally: Police may seize effects for a brief time to investigate and obtain a search warrant, provided they do so with as little delay as reasonably possible. United States v. Van Leeuwen

c. Luggage: Police may detain a person’s luggage on reasonable suspicion, but since such detention may effectively restrain the person from continuing with his travel plans, the detention of luggage is subject to the same limitations as a Terry stop of a person. United States v. Place

1. Rule: you can hold it for a reasonable amount of time, the time it takes to get a warrant. A detention in excess of 90 minutes, is NOT reasonable.

6. PROTECTIVE SEARCH

a. Generally: To justify a protective search, or frisk, for weapons, an officer must be able to point to particular facts from which he reasonably inferred that the individual was armed and dangerous.

b. Scope of Frisk: The scope of the frisk must be a limited intrusion to insure safety, which usually means an initial pat-down of outer clothing to search for weapons, and, if weapons are found, then a more intrusive search.

c. If an initial pat down tells the officer that an item could not be a weapon, he may NOT investigate further simply because the item feels like some other form of contraband. Minnesota v. Dickerson

d. Michigan v. Long ; An officer may search the passenger compartment of an automobile during a Terry stop if he has reasonable suspicion that the suspect is dangerous and the automobile may contain a weapon.

i. However, such a search must be limited to areas which would generally be in the suspect’s control, and which could contain a weapon.

e. U.S. v. Askew ; OK for cops to unzip suspect’s outer jacket for identification purposes during a Terry stop.

7. OTHER BRIEF DETENTION FOR INVESTIGATION

a. Generally: Detention for fingerprinting, without probable cause, that takes place at the stationhouse and involves more than is necessary for the printing, is NOT reasonable under the 4A. OK to subpoena suspect to a grand jury to give fingerprints, doesn’t implicate 5th A.

b. Davis v. Mississippi ; Rape case, and the victim could only supply the general characteristics of the alleged; young, black, male. Prints were found on a window from the rapist’s hand. Big group of black men brought to station, held and fingerprinted w/o warrant.

i. HELD: D’s prints should have been excluded as the fruits of seizure of D in violation of the 4A, but intimated that a detention for such a purpose might sometimes be permissible on evidence insufficient for arrest. No PC to detain or arrest the men in town.

1. Prosecutor should have subpoenaed the men to a grand jury.c. Ds subpoenaed to Grand Jury for fingerprints, voice and handwriting examples. RULE: No

expectation of privacy for a physical characteristic that is on display to the universe. No PC or reasonableness is necessary for a grand jury subpoena b/c this is not a 4A seizure.

d. Dunaway v. New York ; Detention for custodial interrogation requires PC even if it is not technically labeled an arrest b/c it intrudes so severely upon 4A so as to trigger its safeguards against illegal arrest.

i. If an “arrest” is made w/o probable cause, anything that is obtained pursuant to that “arrest” will not be admittable.

e. Bluth: No such thing as “booking on suspicion!” If cops are suspicious they can detain for a brief period of time, but not arrest.

8. ADMINISTRATIVE INSPECTIONS

II. SAFETY INSPECTIONS 1. Safety inspections are allowed and authorities can get warrants on an area wide basis based on

uniformity to inspect.2. Fire inspections: So long as there is an administrative warrant.

III. BORDER SEARCHES 1. Every sovereign state has the right to protect their borders, and Country has same right. Roving

patrols based on race not OK. Fixed checkpoints where they search every 5th car was OK.2. Non-routine strip searches require a real suspicion; cavity searches require a “clear indication.”3. When customs agents “reasonably suspect that the traveler is smuggling contraband in her body

cavities she may be detained so long as is necessary to either verify or dispel the suspicion.”

iv. VEHICLE CHECKPOINTS 1. RULE: roving patrols could stop and search vehicles for illegal aliens only on probable cause.

28

Page 29: Crim Pro Outline 1.1

a. HOWEVER, only Terry-type reasonable suspicion was needed for such patrols engage in the more modest interference with 4A interests which attends the stopping of motorists and inquiring briefly as to their residential status.

2. Delaware v. Prouse ; Absent a reasonable suspicion, the police may not stop individual vehicles for the purpose of checking the driver’s license and the registration of the automobile. Can’t just stop cars w/o a reason.

3. Michigan Dept of State Police v. Sitz ; Upheld fixed sobriety checkpoints absent reasonable suspicion where intrusion is slight.

a. Balanced safety on roads vs. personal intrusion. 4. BUT…Can’t stop cars, through fixed checkpoints, for general law enforcement purposes.

Would be OK if there was a purpose, like finding witnesses to an accident by stopping all cars at a specific given time when accident happened.

V. DRUG TESTING 1. Can the government, generally, make its employees submit to a suspicion-less drug test? NO2. However, depending on the position that the person is hired for,

a. Work w/ illegal drugs then gov’t can test you,b. FBI agent, who has a gun, will generally have to submit to the drug test,c. Airline pilot, Government & RR workers who could be involved in accidents.

VI. SEARCH OF STUDENTS

1. School can search a student w/in limits, don’t need a warrant, but need some suspicion and a policy.

2. Search of student’s locker at school is permissible.3. Bd. of Ed. v. Earls ; Random drug testing does not violate a student’s privacy.

VII. SUPERVISION OF PROBATIONERS AND PAROLES 1. Samson v. California ; (2006) Found that a condition of release can so diminish or eliminate a

released prisoner’s reasonable expectation of privacy that a suspicion-less search by cops would not offend the 4th A.

2. Rule: By virtue of there status alone, probationers do not enjoy the absolute liberty to which every citizen is entitled.

a. BUT…Not OK to search Probationers girlfriend found in Probationers house b/c she does not have the lowered expectation.

VIII. CONSENT SEARCHES

1. Generally: This is the most frequent exception to the warrant requirement; If consent to search is given, then reasonable suspicion, probable cause and a warrant are not necessary.

2. Schneckloth v. Bustamonte (1973); Bustamonte was a passenger in a car owned by his brother that was stopped by the police for an equipment violation. Bustamonte gave the police permission to search the car (they found stolen checks) and now seeks to have the evidence excluded.

a. Rule: For a consent search to be valid the State need only prove consent was voluntarily given and not the result of duress or coercion, either express or implied; the consent giver need not know his right to refuse consent if the consent was voluntarily given.

i. Two Prongs: 1. Consent voluntarily given, and 2. not the result of coercion.b. Rationale: 4th A rights don’t directly implicate the integrity of the guilt or innocence of the

process, whereas confession does. c. If you consent, you can limit the consent. You will want to have the limits recorded so it is

not abused or turned into a he said/she said.

3. FACTORS IN DETERMINING THE VALIDITY OF CONSENT

A. CLAIM OR SHOW OF AUTHORITY

i. A search cannot be predicated on consent when that consent is given only after an officer has asserted that he has a warrant. Claim of warrant in effect tells the occupant he has no right to refuse, it is coercive. Bumper v. North Carolina

B. DENIAL OF GUILT

i. The ready discovery of incriminating evidence after a person has denied guilt does not indicate that this consent must have been involuntary because the reasonable person test presupposes an innocent person. Florida v. Bostick

C. WARNING OF 4TH A RIGHTS i. Don’t have to give 4th A warnings against unreasonable search/seizure and warrant

requirement.

29

Page 30: Crim Pro Outline 1.1

D. WARNING OF 5A RIGHTS

i. The prevailing view is that Miranda warnings need not be given for a consent search to be valid because requesting consent is not likely to elicit an incriminating statement. United States v. LaGrone

E. RIGHT TO COUNSEL

i. A suspect represented by counsel cannot give search consent without the knowledge of his counsel.

F. SCOPE OF CONSENT

i. The scope of a consent search is defined by what the typical reasonable person would understand given the exchange between the suspect and the officer and the expressed object of the search. Florida v. Jimeno

ii. OK to limit the scope of the search, but you will want it recorded.

IX. THIRD PARTY CONSENT

1. Illinois v. Rodriguez (1990); Rodriguez’s former girlfriend used her key to Rodriguez’s apartment to let police in to arrest him. While inside the police discovered evidence of narcotics trafficking. Rodriguez challenged as a 4A violation. In this case, however, there was NO actual authority, she had apparent authority.

a. Rule: 4th A generally prohibits the warrantless entry of a person’s home, whether to make an arrest or to search for specific objects. The prohibition does not apply to situations in which voluntary consent has been obtained, either from the individual whose property is searched, or from a 3rd party who possesses ACTUAL authority over the premises.

b. Actual Authority - rests on mutual use of the property by persons generally having joint access or control for most purposes.

i. Ask: Were the police under a reasonable belief that the 3d person had the apparent authority to give consent?

1. Because many situations which confront officers in the course of executing their duties are more or less ambiguous, room must be allowed for some mistakes on their part. But the mistakes must be those of a reasonable man, acting on facts leading sensibly to their conclusions of probability. They should have asked if she was living there.

c. Objective Standard: Determination of consent to enter must be judged against an objective standard, would the facts available to the officer at the moment warrant a man of reasonable caution in the belief that the consenting party had authority over the premises.

X. WHO HAS AUTHORITY TO GIVE 3D PARTY CONSENT

1. SPOUSE GENERALLY ALLOWED

a. A spouse has presumptive authority to consent to a search of all areas of the homestead; the non-consenting spouse may rebut this presumption only by showing that the consenting spouse was denied access to the particular area searched.

2. ROOMATES W/ SEPARATE BEDROOMS a. OK to consent to search of common area, but probably not the bedroom.

3. PARENT-CHILD

a. The head of the household may give consent to a search of the child’s living quarters.b. A child may not consent to a full search of the parent’s house, but may allow mere entry of

cops.4. LANDLORD-TENANT

a. Generally, LL not allowed to consent to a search of his tenant’s premises.5. HOTEL EMPLOYEES

a. Hotel EEs may not consent to a search of their guest’s room.6. JOINT TENANTS

a. When two or more persons are living as joint tenants, one may consent to a search of the entire dwelling.

7. EMPLOYER-EMPLOYEE

a. The authority of an ER to consent to a search of an EE’s work area depends on the reasonableness of the employee’s privacy expectation (manager w/ desk vs. factory EE).

b. The authority of an EE to consent to a search of his ER’s premises depends on the scope of his authority.

XI. LIMITS OF 3D PARTY CONSENT 1. DEFENDANT’S SPECIFIC INSTRUCTIONS

30

Page 31: Crim Pro Outline 1.1

a. If police know that a person has instructed a 3d party not to consent to a search of his premise, then that 3d party cannot give valid consent for the search.

2. SPECIFIC REFUSAL

a. If the person against whom a search is directed is present and objects to the search, the consent of an absent 3d person will not override that objection.

15. ENTRAPMENT

A. ENTRAPMENT IS A STATUTORY CRIMINAL LAW DEFENSE;i. Question is under what circumstances are we going to allow the defense of entrapment? Did the gov’t

induce the behavior, and if they did induce the behavior, should it have been resisted.ii. Subjective approach (majority); Inducement, focuses on whether the gov’t’s conduct actually caused that

particular D to commit the crime, or whether D was already predisposed to commit the act. 1. If a jury finds that the defendant was entrapped, then the defendant is found not guilty.

iii. Objective approach (minority); focuses solely on the gov’t’s conduct and whether it ‘goes too far’ to be acceptable. Question of law for court.

B. INDUCEMENT i. To be entitled to an entrapment defense, the D must ordinarily show that the gov’t conduct “induced” the D

to commit the crime.

ii. Sherman v. U.S .; (1958) Kalchian, a gov’t informer, induced Sherman into buying drugs for K to use. Sherman was resistant, but K said he was suffering w/drawal. Sherman bought drugs and shared w/ K, Sherman was arrested and claimed defense of entrapment. Here, K was induced. He was not engaging in criminal behavior for personal gain, he thought he was helping out a guy in withdrawal.

1. Subjective Analysis: Entrapment occurs only when the criminal conduct was ‘the product of the creative activity’ of gov’t. To determine whether entrapment has been established, a line must be drawn b/tw the trap for the unwary innocent and the trap for the unwary criminal. Must examine the conduct of the gov’t agent and the conduct and predisposition of the accused using the facts of the case.

iii. The Meaning of Inducement: Implant & Induce. For the entrapment defense to be available, the gov’t must “implant in the mind of an innocent person the disposition to commit the alleged offense and induce its commission.”

1. Inducement = opportunity plus something else, typically excessive pressure by the gov’t upon the D or the gov’t’s taking advantage of an alternative, non-criminal type of motive.

C. THE TESTS OF ENTRAPMENT

I. SUBJECTIVE APPROACH 1. United States v. Russell (1973); Federal courts. A man convicted of unlawfully manufacturing

meth sought to challenge his conviction on the ground that gov’t entrapped him by providing him with an essential (legal) ingredient in the manufacturing process. Principal element in the defense of entrapment was the D’s predisposition to commit the crime. However, the defense of entrapment was not intended to give the federal judiciary a “chancellor’s foot” veto over law enforcement practices of which it did not approve. Notwithstanding the temptation of committing the crime, was the defendant predisposed of committing the crime? Here the answer was YES. He was predisposed and he did it for profit, so no entrapment defense here.

a. Holding and Rule: i. Inducement: Gov’t conduct here stops far short of violating that “fundamental

fairness, shocking to the universal sense of justice,’ mandated by the DPC & 5th A. Predisposition: D would have committed the crime anyway, so entrapment defense is unavailable.

ii. Rule from Sorrells1. The entrapment defense prohibits law enforcement officers from

instigating a criminal act by persons “otherwise innocent in order to lure them to its commission and to punish them.”

2. Thus, the intent or predisposition to commit the crime is a key element.

a. Predisposition: D would have committed the crime anyway, so entrapment defense is unavailable.

iii. Rule from Sherman1. To determine whether entrapment has been established, a line must be

drawn between the trap for the unwary innocent and the trap for the unwary criminal.

31

Two Step Process:

1. Did the government induce the defendant to commit the crime

2. Is the particular defendant predisposed to commit the crime

If the defendant was predisposed to commit the crime, the entrapment defense is UNAVAILABLE

Page 32: Crim Pro Outline 1.1

D. PREDISPOSITION

i. Definition: Predisposition is a state of mind which readily responds to the opportunity furnished by the officer or his agent to commit the forbidden act.

ii. Jacobson v. United States (1992); Bought kid porn when it was legal. Cops tried to get D to buy child pornography for over 2 ½ years after law against child porn went into effect.

1. Holding and Rule:a. Where the government has induced an individual to break the law and the defense of

entrapment is at issue, as it was in this case, the prosecution must prove beyond a reasonable doubt that the defendant was predisposed to commit the criminal act prior to first being approached by Government agents. Here, he bought 1st magazine when it was legal, and stopped when laws changed.

b. Law enforcement officials go too far when they implant in the mind of an innocent person the disposition to commit the alleged offense and induce its commission in order that they may prosecute.

iii. Cop dressed as bum asleep in Central Park w/ $20 hanging out of his pocket, thief takes money. Entrapment? NO.

iv. Cops open 2nd hand shop and advertises no questions asked for goods. They take pics of everyone that brings stuff in, get the goods w/ serial numbers and give marked cash. Arrest all people. People say the only reason they stole the stuff was because Cops were there to buy it. Entrapment? NO.

v. To Catch a Predator TV show – Entrapment? NO, but it is very difficult to find a crime that the men have committed.

vi. United States v. Kelly (1983) (D.C. Cir.) Not in book. Con man helped FBI target Congress, b/c Congress could pass private bills of citizenship immediately for money. A congressman who was convicted of taking bribes sought to challenge his conviction on the ground that the FBI’s conduct in hatching and carrying out the entire scheme was a violation of due process.

1. Issue: Should the FBI be able to troll in Congress like this?2. For a D to successfully challenge his conviction based on law enforcement’s over-involvement in

a criminal enterprise, he is required to show that the conduct of the government’s agents involved coercion, violence, or brutality.

3. S.C. said NO due process violation here, OK for the FBI to troll (but they have not done this since).

16. POLICE INTERROGATION AND CONFESSIONS

a. Problem: given enough time and pain, Bluth could get anyone of us to confess to killing Kennedy. What is the value of a confession that is obtained through torture? Some will be accurate, others won’t. We want a confession to be voluntary, produced through a conscience decision to tell what happened.

b. MCNABB-MALLORY RULE i. 1940 - S.C. said you have to bring the D in front of the judge as soon as reasonably possible. The S.C.

held that, while state law enforcers were held to minimal 14A ‘due process’ standards, the S.C. could set higher standards for federal law enforcement. McNabb v. United States

1. For example, confessions obtained while illegally detained are inadmissible, confession obtained after protracted detention inadmissible. Mallory v. U.S.

ii. Gradually, S.C. decision increased standards for STATE police by strengthening the right to counsel and privilege against self-incrimination.

C. PRIVILEGE AGAINST SELF-INCRIMINATION

i. Massiah v. United States ; (1964) Drug dealer who pled not guilty, hired lawyer, and was released on bail and questioned by undercover police informer in car. Court held D’s incriminating statements could not be used against him at his trial – 6th A right to counsel.

ii. Later cases held that persons who become the “focus” of criminal investigation have the right to see their lawyer, OR AT LEAST BE TOLD OF THEIR CONSTITUTIONAL RIGHT TO REMAIN SILENT.

1. Escobedo v. Illinois ; (1964) after murder suspect was not permitted to see his lawyer, his incriminating statements obtained by police-arranged confrontation with codefendant was INADMISSIBLE.

iii. However, the police are still allowed to ATTEMPT pretrial interrogation, though the suspect may refuse to answer.

D. MIRANDA IS BORN

32

Page 33: Crim Pro Outline 1.1

i. Miranda v. Arizona (1966); A rapist who confessed during his custodial interrogation appeals his conviction, contending he was not informed of his constitutional privilege against self-incrimination and right to counsel.

1. Rule: Confessions produced by custodial interrogation are INADMISSIBLE unless the suspect was informed of:

a. His right to silence and the consequences of waiving that right, b. His right to retained or appointed counsel, ANDc. The suspect waived those rights knowingly and intelligently.

If custodial interrogation is made without these procedural safeguards, all statements obtained are INADMISSIBLE

2. The prosecution may not use statements, whether exculpatory or inculpatory, stemming from custodial interrogation of the D unless it demonstrates the use of procedural safeguards effective to secure the privilege against self-incrimination. Telling the suspect what their rights are will provide balance.

3. Custodial interrogationa. By custodial interrogation, we mean questioning initiated by law enforcement officers

after a person has been taken into custody or otherwise deprived of his freedom of action in any significant way.

b. Sure signs of custody: Handcuffs.4. Procedural Safeguards

a. Unless other fully effective means are devise to inform the accused person of their right to silence and to assure a continuous opportunity to exercise it, the following measures are required;

b. A waiver of these rights is possible, but only if made voluntarily, knowingly and intelligently Johnson standard voluntary relinquishment of a known right.

i. How is a waiver accomplished must be an EXPRESS STATEMENT that the right has been waived.

1. If a police officer is claiming that a waiver occurred, then he will have the burden of proving that it occurred.

II. SILENCE WILL NOT BE A PRESUMED WAIVERc. What then, is an acceptable method of waiver?

i. Signed waiver, though this is difficult to get .5. No questioning may occur if the person, in any way, indicates that he is not waiving the

rights.a. If the suspect first answers some questions or volunteers some information before

invoking his right to remain silent, he is NOT deemed to waive his rights for any post-invocation statements.

b. Suspect’s failure to request an attorney is not deemed a waiver of the right to counsel, unless he is informed of the right, and then chooses to confess anyway.

c. Remember, ON THE SCENE questioning is just fine b/c you are NOT in custody, this is honest and general investigation.

6. Suspect asserts attorney requesta. If the defendant has asserted the right to an attorney, the police may not continue any

interrogation whatsoever, UNLESS, the defendant approaches the police voluntarily to continue the interrogation.

i. At the time the suspect indicates his request of an attorney, the suspect must have an opportunity to confer with the attorney and have him present during any subsequent questioning.

b. If the suspect indicates he wants an attorney before speaking with police, but cannot obtain one, the police cannot question him until one is provided.

7. NOTE These rights must only be given once.8. On the scene interrogation is okay without giving the warning. 9. Voluntary statements are admissible, no need to give warning (guy walks into police station and

admits killing wife, admissible even w/o prior Miranda warning). 10. PROBLEM: Miranda does not deal with prolonged interrogation. Miranda does not deal w/ cops

lying to get a confession. Thus, does not balance the power of custodial interrogation anymore. 11. SOLUTION: Video tape all custodial interrogations.

ii. Congress’ response 1. Omnibus Crime and Control of 1968 Act § 3501 = Miranda is repealed

33

Miranda Warnings Prior to the questioning, the person must be warned that he has a right to remain silent, that any statement he does make may be used as evidence against him, and that he has a right to the presence of an attorney, either retained or appointed

Waiver of Right

Assertion of Right of Attorney

Page 34: Crim Pro Outline 1.1

a. Going back to the rule where the question was whether the confession was voluntary as a matter of fact considering the “totality of circumstances.”

2. The problem is that if Miranda is a Constitutional issue, a statute cannot repeal a Court decision must Amend the Constitution.

3. A line of S.C. cases held that Miranda was NOT a constitutional right, but the Court never enforced § 3501 b/c they were afraid what the S.C. would do.

iii. Post- Miranda Supreme Court Case Law 1. Police may warn suspects of their right to silence and counsel through warnings other than

the standard Miranda warning. 2. Admissions obtained with defective Miranda warnings are still admissible to impeach.

a. However, confessions obtained after an illegal ARREST are inadmissible regardless of any warnings.

3. If a suspect refuses to answer questions, the police must stop questioning, but may restart later—under some circumstances.

a. However, if the suspect demands a lawyer, the police CANNOT re-question him before providing one.

4. If police obtain admissions without giving proper Miranda warnings, but then give proper warnings, any admissions obtained after the proper warning ARE admissible.

5. Even jailhouse questioning may not be “custodial interrogation.”a. Oregon v. Mathiason not custodial where suspect voluntarily went to police station for

questioning.

E. APPLYING AND EXPLAINING MIRANDA i. OK TO EXPLOIT SUSPECTS’ IGNORANCE

1. Confessions given because the suspect did not understand the confession would be incriminating ARE ADMISSIBLE.

a. State v. McKnight ; suspect confesses, believing oral confession was inadmissible.

II. DEFECTIVE WARNINGS

1. Courts will waive minor defects in police officers’ field recollections of the Miranda warnings, as long as the meaning is clear and correct.

iii. ADDITIONAL REQUIRED WARNINGS 1. Right to silence, currently the suspect is not told “if you choose to remain silent, your silence will

not be used against you as evidence to suggest that you committed a crime b/c you refused to speak.”

2. Suspect not told that if he chooses to talk to the police at the outset, he may terminate the conversation at any time.

3. Suspects need not be told of the proposed subject of the interrogation.a. Colorado v. Spring ; when suspect questioned for gunrunning confessed to murder, the

confession is admissible, even though police never revealed they also suspected him of murder.

iv. “CUSTODY” VS. “FOCUS” 1. Escobedo’s procedural safeguards were triggered when the person became the focus of police

investigation.a. However, the “focus” test was rejected in favor of custody test; Beckwith v. United

States; Stansbury v. California.

v. “CUSTODY” AND “CUSTODIAL INTERROGATION” 1. “Custody” for Miranda purposes is an objective test – how would reasonable people in the

suspect’s situation have perceived their circumstances?2. Most courts hold that police’s on-the-street questioning is not “custodial,” absent special

circumstances (e.g., arresting at gunpoint or forcibly subduing a suspect).Traffic stops are NOT custodial interrogations.

3. Hostage case, cops will talk to suspect, cops surround the suspect but he is not in custody and does not get Miranda warnings.

4. Even jailhouse questioning may not be custodial, as where the suspect voluntarily goes there to answer questions.

34

Remember, rationale of Miranda is to prevent coerced confession while in custody. Magic Words, “I don’t want to talk to you and I want a lawyer.”

Page 35: Crim Pro Outline 1.1

a. Oregon v. Mathiason ; voluntary jailhouse interview not “custodial.”b. California v. Beheler ; even when “voluntarily” accompanied by police.c. Berkemer v. McCarty ; roadside questioning incident to routine stop not “custodial.”

Traffic stops are not custodial interrogations.

5. Suspects age is relevant to “Custody” Issuea. J.D.B. v. North Carolina ; (2011); 13-yr old in office w/ 2 cops and confesses to crime,

not given Miranda warnings b/c not in “custody” (reasonable adult standard). Kid wants confession suppressed.

i. Held: S.C. determined a child's age properly informed the Miranda custody analysis since, (1) a reasonable child subjected to police questioning will sometimes feel pressured to submit when a reasonable adult would feel free to go, and courts can account for that reality without doing any damage to the objective nature of the custody analysis, and (2) a child's age differed from other personal characteristics that, even when known to police, have no objectively discernible relationship to a reasonable person's understanding of his freedom of action.

vi. INTERROGATION 1. Rhode Island v. Innis (1980); After murder suspect in police custody requested counsel, one

officer remarked the lost murder weapon might injure nearby children, which induced the suspect to locate the weapon. Issue is whether D was “interrogated” in violation of Miranda.

a. Rule: Even if suspect is in police custody, police officers’ statements are not deemed ‘interrogation’ unless they are either express questions or functionally equivalent statements which are reasonably likely to elicit an incriminating response , and in fact elicit such a response.

i. The concern for the Court in Miranda was that the “interrogation environment” created by the interplay of interrogation and custody would “subjugate the individual to the will of his examiner” and thereby undermine the privilege against compulsory self-incrimination.

ii. The definition of interrogation can extend only to words or actions on the part of police officers that they should have known were reasonably likely to elicit an incriminating response.

1. Reasonably is gauged by average suspects’ perceptions, not the police’s intent.

2. Arizona v. Mauro ; Cops record wife’s conversation with husband (who had been given Miranda warning) suspected of killing son. Husband was in interrogation room, but was talking freely to his wife w/ recorder in plain sight. S.C. says OK.

a. Remember, Miranda meant to prevent gov’t from using the coercive nature of confinement to extract confession that did not happen here.

vii. IN JAIL AND POLICE “PLANTS”1. Miranda warnings are NOT required when the suspect is unaware he is talking to a police agent.

a. Illinois v. Perkins ; police enlist convict and undercover officer posing as convict to elicit voluntary confession. Doesn’t implicate Miranda rationale to protect against coercive police environment and forced confession.

2. Different from Messiah where cops can’t send an informant out to talk to suspect, here in jail there are no rights to privacy.

viii. ROUTINE “BOOKING QUESTION” EXCEPTION 1. Police may ask standard booking questions—name, address, height, weight, eye color, birth date,

SS # and age—without Miranda warnings. Can’t ask a question like “how old were you in 1982?” b/c this question is meant to see how sober you are.

a. Pennsylvania v. Muniz ; DUI suspect’s inability to recall basic information is admissible as evidence of intoxication.

IX. “PHYSICAL” OR “DEMEANOR” EVIDENCE VS. EVIDENCE OF SILENCE 1. U.S. v. Velarde-Gomez (2001); Prosecutor told jury D had no emotional or audible reaction when

cops informed him they found 63 pounds of pot in gas tank. Court said gov’t may not comment on D’s post-arrest silence at trial.

x. RESPONSE TO OFFICER’S QUESTIONS USED AS INCRIMINATION 1. Hiibel v. Sixth Judicial Court (2004); Disclosure of P’s name presented no reasonable danger of

incrimination in criminal trial.

35

Page 36: Crim Pro Outline 1.1

xi. PUBLIC SAFETY EXCEPTION 1. New York v. Quarles (1984); After police arrested an armed suspect who hid his gun, they asked

him where the gun was before reciting Miranda. At trial, the suspect moved to exclude the gun and his answer. No claim that D’s statements were actually compelled by police conduct which overcame his will to resist.

a. Public Safety Rule: Police may ask questions reasonably necessary to ensure public safety without reciting Miranda warnings, and suspects’ replies are admissible.

xii. TORTURE 1. If you torture someone to get vital information, to save someone’s life is it admissible? Current

controversy in war on terrorism. Is it OK to torture to get the information we want/need, even from people we won’t try in court? Public safety exception?

2. Experts tell us kindness works better than violence.

xiii. WAIVERS 1. PROVING WAIVER

a. Remember, waiver won’t be presumed from a silent record.b. The Supreme Court stopped short of requiring police to tape administered Miranda

warnings to prove confessors waived them.c. Similarly, most states do not require taping, and accept officers’ uncorroborated

testimony.

2. IMPLIED WAIVER a. Mere silence does not indicate waiver.b. HOWEVER, silence and conduct indicating the suspect understands his rights but

chooses to waive them is sufficient, though the police bear the proof burden (where suspect refused to sign waiver but nonetheless indicated that he was willing to talk); North Carolina v. Butler.

c. Approximately 80% of custodial suspects waive their rights. Many believe remaining silent will make them look guilty, and they should try to persuade the cops they are not guilty by talking.

d. Berghuis v. Thompkins (2010); After Miranda given D was silent for 3 hours, but eventually statement made and D convicted.

i. Rule: If an accused makes an ambiguous statement or NO statement concerning right to counsel, the cops are not required to end the interrogation or ask questions to clarify whether D wants to invoke his Miranda rights.

3. “QUALIFIED” WAIVER a. Sometimes, suspects refuse to confess or make written confessions, but are willing to

discuss the crime or make an oral confession, apparently because they believe (mistakenly) that oral statements are inadmissible. Such oral confessions or incriminating statements ARE admissible, even if the suspect failed to understand this.

b. Or a suspect who won’t talk w/ a tape recorder, but WILL talk without one. STUPID, you are much better off, if you are going to talk at all, to have the conversation recorded.

c. Connecticut v. Barrett ; Rapist demanded counsel before signing anything or putting anything in writing, but then volunteered incriminating information and it was admissible.

xiv. INVOKING MIRANDA 1. Suspects’ asking to consult a non-lawyer (e.g., probation officer, priest, friend) is NOT deemed an

invocation of the right to counsel. a. Fare v. Michael ; juvenile’s request to see probation officer did not invoke Miranda. The

admissibility of the statements on the basis of waiver turned on the “totality of the circumstances surrounding the interrogation” (age, experience, background and IQ).

XV. PROCEDURE AFTER INVOCATION 1. RIGHT TO SILENCE [5A right against self-incrimination]: Once suspects assert their right to

remain silent, police must:a. Stop questioning immediately;b. Wait for a significant period without questions of any type;

36

Remember, suspect must INTELLINGENTLY and KNOWINGLY relinquish his rights.

Page 37: Crim Pro Outline 1.1

c. Recite Miranda before resuming questioning.2. Michigan v. Mosley ; after robber invoked silence, he may be questioned about another crime two

hours later after new warnings given.

3. RIGHT TO COUNSEL [6A right to representation]a. When a suspect asserts the right to counsel, the police must seize questioning and the

police may not try again in the future. Can’t come back two hours later and try again, unless they provide him with a lawyer. If they provide atty, they can question him again later w/ atty present on the same crime.

i. Arizona v. Roberson; Once a suspect asserts his Miranda-Edwards right to counsel, the cops cannot even initiate interrogation about crimes other than the one for which the suspect has invoked his rights. But if he has been indicted so that the D knows the charge against him, the cops can approach him to ask if he will speak about a different crime.

4. Maryland v. Shatzer (2010); Inmate invokes right to counsel and is questioned 2 ½ years later on same crime, makes incriminating statement, and seeks to have statement w/held.

a. Rule: a 14-day break in custody eliminates Edward’s protection.

5. INITIATING

a. Generally, if after a suspect requests counsel, a dialogue ensues with the police, the police may be deemed not to have “initiated” it, thus avoiding Miranda’s requirement. Involves routine inquires incident to the custodial relationship.

i. Oregon v. Bradshaw ; DUI/vehicular homicide confession obtained w/o atty after suspect asked police “What is going to happen to me now?” is admissible.

6. ASSERTING THE RIGHT TO COUNSEL

a. To invoke the Edwards right, the suspect must EXPLICITLY and UNAMBIGUOUSLY invoke the right. Davis v. United States.

i. Rule in Miranda is that a suspect is entitled to the assistance of counsel during custodial interrogation.

ii. Rule in Edwards that if the suspect invokes the right to counsel at any time, the cops must immediately cease questioning him until atty present.

iii. The Court was unwilling to create a third layer of protection to prevent police questioning when the suspect might want a lawyer. Unless the suspect actually requests an attorney, questioning may continue.

b. Gender Differencesi. Should the court recognize that there might be gender differences in the

manner in which the right is typically asserted?1. Justice O’Connor has refused to recognize the difference in Davis.

7. Montejo v. Louisiana (2009); When court appoints atty for indigent D in the absence of any request on his part, there is no basis for a presumption that any subsequent waiver of the right to counsel will be involuntary. B/f suspect met lawyer he went w/ cops to locate murder weapon and wrote letter of apology to family. Here, state invoked his right for him, he did not invoke his own rights.

a. This won’t apply in most States, this is a problem particular to Louisiana.b. Messiah says you can’t talk to a suspect on the street once he has been indicted and has a

lawyer.

8. OFFENSE-SPECIFIC RIGHT TO COUNSEL

a. If suspect requests counsel in response to a charged crime (this means he has been arraigned and knows the charge), then police may question the suspect about unrelated or un-charged crimes without counsel.

i. McNeil v. Wisconsin ; suspect who invoked counsel for charged robbery may be questioned about separate murder.

XVI. POLICE DECEPTION ABOUT COUNSEL 1. Moran v. Burbine (1986) After the police failed to tell a suspect that a lawyer had been appointed

for him, and falsely told the lawyer the suspect would not be interrogated, the suspect confessed. Miranda warnings were given and the suspect’s sister contacted an attorney. The attorney called the police station and the police said that the suspect wouldn’t be questioned in the morning; however, the suspect was questioned that night.

37

Page 38: Crim Pro Outline 1.1

a. Rule: Miranda issue is that the suspect did not ask for an atty, so he doesn’t get an atty. Most state courts don’t like this case and have adjusted their constitutions accordingly, saying that it is NOT ok to lie to a defense lawyer in order to obtain confession.

i. The Court concludes that because the suspect’s voluntary decision to speak was made with full awareness and comprehension of all the information Miranda requires the police to convey, the waivers were valid.

ii. NOTE: Many states have refused to accept this broad decision and have read the right to counsel more narrowly to say that a police officer may NOT lie to a defense lawyer to obtain a confession

XVII. PSYCHIATRIC EXAMINATION 1. In Ohio, three circumstances where a psychiatric exam will be required:

a. D pleads not guilty by reason of insanity, D claims they are not competent to stand trial, or if the D asserts a battered woman defense and is going to assert expert testimony.

b. Examiner cannot testify to the things told to her, but can only relate their professional opinion as to whether the D is insane or is a battered woman.

2. Use at penalty stagea. If a psychiatric exam’s results are to be admissible in a trial’s penalty stage, the

psychiatrist must obtain the defendant’s lawyer’s permission for the exam, and the defendant must be read Miranda; Estelle v. Smith

i. Just as the 5A prevents a criminal defendant from being made the deluded instrument of his own CONVICTION, it protects him as well from being made the deluded instrument of his own EXECUTION.

XVIII. MENTALLY RETARDED

1. Many tests have shown that mentally retarded person will not understand Miranda warnings, no matter how it is given.

XIX. FEDERAL APPEALABILITY OF MIRANDA

1. Withrow v. Williams (1993); After a murderer was convicted using a confession obtained without Miranda warnings, he attempted to appeal to the federal courts, on habeas corpus grounds.

a. Rule: Miranda claims are subject to federal appeal on habeas corpus. A habeas corpus claim may be brought on a 5A claim (right to silence) and a 6A claim (right to atty).

i. Miranda safeguards a fundamental trial right.ii. Miranda serves a value necessarily divorced from the correct ascertainment of

guilt.iii. Eliminating review of Miranda claims would not significantly benefit the

federal courts in their exercise of habeas jurisdiction.

f. 18 U.S. § 3501: 1968 Congressional legislation that repealed Miranda by giving the judges the ability to deemed whether a confession was voluntary given the “totality of the circumstances.” U.S. Atty General never wanted this legislation enforced.

G. STATUTORY LIMITATION OF MIRANDA

i. Dickerson v. United States ; (2000) S.C. considered whether Miranda was a constitutional holding, and thus not subject to being superseded by Congressional legislation, originating from the granting of Dickerson’s motion to suppress because he had not received Miranda warnings before interrogation.

1. Rule: S.C. announced Miranda is a constitutional rule, which Congress may not supersede legislatively.

H. PRACTICAL EFFECTS OF MIRANDA

I. EMPIRICAL STUDIES

1. Most empirical studies show prosecutors, judges, and police officers feel that Miranda requirements have not hampered law enforcement significantly.

2. However, some feel police evade Miranda (e.g., by reading their Miranda right but then coercing them into waiving them by implying they will receive harsher sentences if they request a lawyer or don’t confess).

I. THE DUE PROCESS AND VOLUNTARINESS TEST REVISITED

I. GENERALLY

1. Miranda superceded the older test for admissibility of confession, which held that due process was not violated if the confession was “voluntary” considering the totality of the circumstances

a. However, the “voluntariness” test is still applicable when suspects waive their Miranda rights and answer police questions, since the answers must still be “voluntary”

38

Page 39: Crim Pro Outline 1.1

i. Further, the “voluntariness” test is applicable in situations outside Miranda’s scope—e.g., non-custodial police interrogation, custodial questioning or threats by private citizens, use of confession to impeach, or the use of the “fruits” of the confession

b. Confessions will be used for impeachment purposes, so there may be an incentive to press for a confession even if it will not be admissible for the case in chief

J. MIRANDA, THE PRIVILEGE AGAINST COMPELLED SELF-INCRIMINATION

i. Chavez v. Martinez (2003); Guy was in the hospital after being shot and police continued to question him, even as he was complaining of belief of imminent death; Miranda warnings were not given; criminal charges were not brought, but guy filed a § 1983 civil action against the police officer.

1. Rule: Mere coercion does not violate the text of the Self-Incrimination Clause absent use of the compelled statements in a criminal case against the witness. Section 1983 doesn’t provide remedies for violations of Miranda, P seeking redress through § 1983 must establish the violation of a federal constitutional or statutory right.

K. IS PHYSICAL EVIDENCE OR A “2ND CONFESSION” DERIVED FROM A FAILURE TO COMPLY WITH MIRANDA RULES ADMISSIBLE?

i. “Fruit of the poisonous tree” Doctrine: evidence otherwise admissible but discovered as a result of an earlier violation is excluded as tainted, lest the law encourage future violations. Wong Sun v. U.S.

ii. Oregon v. Elstad (1985); Cops went to Elsatd’s home w/ warrant for his arrest, then knew they were going to arrest him, but they sat down and talked to him first. Elstad told them about the crime, and then cops arrested him and gave him Miranda and asked him to repeat his confession.

1. S.C. declined to apply the “poison fruit” doctrine to a “second confession” following a confession obtained w/o giving D the Miranda warnings. Thus, secondary evidence derived from a Miranda violation need not, and should not, be suppressed as tainted.

a. This formed the basis for a change in police interrogation manuals, so cops started to get confessions w/o warnings, and then give warnings and ask to repeat confession.

iii. U.S. v. Patane (2004); D interrupted cop giving D Miranda, D said he knew his rights and no further warning was given. D told cop location of gun and was convicted. S.C. plurality said Miranda is a prophylactic rule employed to protect against violations of the Self-Incrimination Clause, however, Miranda is NOT implicated by the admission into evidence of the physical fruit of a voluntary statement. Potential violations occur, if at all, only upon the admission of unwarned statements into evidence at trial.

iv. Missouri v. Seibert (2004); Cops get confession w/o Miranda warning, then give Miranda and lead suspect through previous confession again. S.C. holds that a statement repeated after a warning in such circumstances is inadmissible. The Court does not like the fact that the police department was training officers on how to deliberately circumvent Miranda.

v. Ohio Rule: In Ohio, stopped for speeding and cop smelled pot on guy (not enough probable cause to put guy in custody or search the car). Guy put in back of locked cruiser and cop told him he was going to search the car and asked if there were any drugs w/o Miranda. Guy said there was a bowl in the trunk. Cop found bowl and gave Miranda. Guy said they were his drugs.

1. Ohio S.C. found guy was in custody and should have been given Miranda warnings, so 1st statement excluded, physical evidence excluded, 2nd statement excluded. Custody is found if there is a deviation from the normal traffic stop. Removing an individual from a car is sufficient enough to trigger custody.

Professor: Technology can make Miranda obsolete. A simple rule that allows police recording from begin of encounter until the end of the encounter. Craft a rule that says we will only allow a confession from which the fact-finder decides is voluntary.

L. POLICE DECEPTION AFTER SUSPECT WAIVES RIGHTS

I. USE OF SYMPATHY 1. If the police use deception to obtain statements AFTER a suspect waives his Miranda rights, the

resulting statements are NOT necessarily INADMISSIBLE unless the deception was EXTREME.a. Miller v. Felton ; post-waiver confession ADMISSIBLE after interrogation faked

sympathy, lied that victim was alive, and suggested killer needed treatment rather than imprisonment. S.C. held that under the “totality of circumstances” of this case, Miller’s confession was voluntarily given. Court considers how manipulative the cop’s tactics are, is the confession a product of free choice.

39

Page 40: Crim Pro Outline 1.1

2. U.S. v. LeBrun ; (2004); Murder confession obtained after cops made a promise not to prosecute. Court held confession voluntary and admissible.

3. However, some cases hold that confessions are inadmissible if the police’s show of sympathy suggests a promise of lenient treatment, or if the police manufacture evidence (fake DNA results).

II. THREATS OF VIOLENCE 1. Arizona v. Fulminante ; FBI officer (jail plant) says that the suspect was going to get his ass

kicked in jail, but that he would protect him if he told truth. Court held that the confession was coerced when there is a credible threat of violence. A finding of coercion need not depend upon actual violence by a government agent; a credible threat is sufficient.

III. CAUSATION 1. Confessions will not be deemed ‘involuntary’ unless they involve coercive police acts that are

causally related to the confession. Confessions prompted by ‘God’ are admissible and voluntary.2. Colorado v. Connelly ; Insane murderer’s confession prompted by belief God commanded him to

confess to police in Denver about 9 month old killing. Court held that absent police conduct causally related to the confession, there is simply no basis for concluding that any state action has deprived a criminal defendant of due process of law.

M. WHEN THE RIGHT TO COUNSEL ATTACHES i. Brewer v. Williams (Williams I) ; (1977) Girls legs seen sticking out of carpet roll. Police elicited

statements during a long automobile ride from a very religious mental patient accused of murder, and represented by counsel, resulting in body being found. Christian burial speech case.

1. Rule: The right to counsel is violated if incriminating statements are obtained from the accused after judicial proceedings have been initiated and counsel retained.

a. Christian burial speech had been tantamount to interrogation,b. The Rule under Massiah is that once adversary proceedings have commenced against

individual, he has a right to legal representation when the government interrogates him,c. Further, the state falls far short of its obligation of proving that Williams met the Johnson

voluntary relinquishment standard (intentional relinquishment or abandonment of a known right or privilege).

N. “NO-CONTACT” RULE i. Generally

1. Most legal ethics codes have a “no contact” rule, whereby lawyers are not to communicate with a party they know is represented by counsel.

2. While cases hold this rule prevents prosecutors from speaking with represented criminal defendants, courts rarely suppress statements elicited by such contact.

O. QUESTIONING FOR UNRELATED REASONS i. If the government continues questioning the suspect after he demands counsel for legitimate reasons

unrelated to evidence-gathering—e.g., to prevent other crimes—the statements are INADMISSIBLE at the suspect’s trial.

1. Maine v. Moulton ; (1985) police may question suspect to prevent his confederates from killing witness, but cannot use the evidence against him.

P. USE OF SECRET/UNDERCOVER AGENTS i. Generally people who are planted inside a jail

1. Prior case law suggested that if government-planted agents overhear suspects’ incriminating statements, they are INADMISSIBLE, even if the agent was “passive” and never initiated conversation with the suspect. U.S. v. Henry

2. However, this rule has been modified, and these statements are admissible absent a showing of coercion.

3. Kuhlman v. Wilson ; Since the 6A is not violated whenever—by luck or happenstance—the State obtains incriminating statements from the accused after the right to counsel has been attached. Rather, the defendant must demonstrate that the police and their informant took some action, beyond merely listening, that was designed deliberately to elicit incriminating statements.

a. RULE: a passive secret agent is ok (overhears D’s conversation), but an active agent is not allowed (initiates conversation w/ D).

Q. RIGHT TO COUNSEL FOR UNCHARGED OFFENSES RELATED TO THE CHARGED OFFENSE i. Texas v. Cobb (2001); Cobb, charged with burglary and represented by counsel, confessed to an uncharged

murder and thereafter claimed his right to counsel attached for the murder as well.1. Rule: The 6A right to counsel does not bar the police from interrogating respondent about

crimes that he has not been formally charged with. 6A is offense specific, w/in the four

40

Page 41: Crim Pro Outline 1.1

corner of the charging document (Bluth thinks it should be event specific). This is a tool to get around Edwards. The tool is for police to 1) UNDERCHARGE, 2) provide constitutional rights required, and then 3)interview for the main charge. Once information is gathered then charge D.

17. LINEUPS, SHOW-UPS AND OTHER PRE-TRIAL IDENTIFICATION PROCEDURES

A. THE DANGERS OF EYEWITNESS IDENTIFICATIONS AND RELATED CONSTITUTIONAL CONCERNS

i. Eyewitness identification can be dangerous: 1. They are notoriously unreliable, and any prejudice in a lineup or other identification procedure,

such as a subtle suggestion by the police, may taint witness identification.2. Furthermore, a pretrial lineup is particularly important because it is unlikely that a witness will go

back on his word later after picking someone out at a lineup.

b. United States v. Wade (1967); A man charged with robbing a bank was subjected to a lineup without his counsel present. –

i. Rule: Conducting a post-charge lineup w/o the D’s counsel present violates the D’s right to counsel.1. The accused’s inability effectively to reconstruct at trial any unfairness that occurred at the line-up

may deprive him of his only opportunity meaningful to attack the credibility of the witness’ courtroom identification – critical stage!

ii. Rationale: Since it appears that there is grave potential for prejudice, intentional or not, in the pretrial lineup, which may not be capable of reconstruction at trial and since presence of counsel itself can often avert prejudice and assure a meaningful confrontation at trial, there can be little doubt that for Wade the post-indictment lineup was a critical stage of the prosecution at which he was “as much entitled to such aid of counsel as at the trial itself.

iii. Solution: If cop transport V to line-up, make sure cop doesn’t know who suspect is. Or, show the suspects in the line-up ONE AT A TIME!

C. WHAT ABOUT IN-COURT IDENTIFICATION:i. Wong Sun Test Whether, granting establishment of the primary illegality the evidence to which instant

objection is made has been come at by exploitation of the illegality OR instead by means sufficiently distinguishable to be purged of the primary taint.

1. Basically, if there is an independent means to support the in-court identification (V has mental picture in mind that evolved during crime) then the in-court identification will come in.

2. Lower Courts generally have not had a hard time finding an independent basis for in-court identifications.

II. NOTES AND APPLICATION

1. Role of Counsel at Lineup: Lower Courts and some commentators have generally agreed that counsel should play a passive role, and should not make the lineup into an adversarial proceeding.

2. Because lineups and handwriting samples do not involve the privilege against self-incrimination, prosecutors are permitted to comment at trial on a party’s refusal to participate in either procedure.

iii. Wade-Gilbert Rule (1967) S.C. decided that there is no right to have counsel present when taking a handwriting sample (or other exemplar) from a D, b/c not a “critical” stage of the criminal proceedings.

1. Thus, some pre-trial identification procedures must be attended by D’s atty while others need not.2. The Court also found that a lineup conducted in an auditorium in which 100 or so witnesses made

identifications of the D in the presence of one another was IMPROPER because of the dangers of improper suggestion that existed.

3. The Court has held that its decision in Wade was not retroactive and that a due process totality of the circumstances test would apply to identifications that took place pre-Wade

D. THE COURT RETREATS: KIRBY & ASH

i. Kirby v. Illinois ; (1972) Defendants arrested for robbery, but not formally charged, were identified at the police station by the victim without their counsel present.

1. Rule: A criminal defendant’s rights to counsel does not attach for purposes of witness identification UNTIL the government initiates criminal proceedings against the D - whether by way of formal charge, preliminary hearing, indictment, information, or arraignment.

a. But, when do adversarial proceedings begin?i. Each state will have the right to determine this on its own.

41

Page 42: Crim Pro Outline 1.1

ii. Note, that Terry would allow the police to stop and detain a person on the street until the victim could identify him of committing the crime these would not be adversary proceedings (laptop stolen hypo in class).

b. So Kirby did not have the right to have counsel present at the police station when a line-up was taken BEFORE the he had been indicted or otherwise formally charged with any criminal offense.

c. Due Process, per Stovall, forbids a lineup that is unnecessarily suggestive and conducive to irreparable mistaken identification.

ii. United States v. Ash ; (1973) A criminal D objected to the government’s use of a photographic lineup without the presence of defense counsel.

1. Rule: A criminal defendant has no right to have counsel attend a photographic lineup conducted by the government.

E. DUE PROCESS AND OTHER LIMITATIONS i. Stovall v. Denno ; Hospital deathbed ID OK b/c of totality of circumstances. S.C. held that even if the

Wade right to counsel does not apply in a given case, a D can still have an identification excluded when it is shown that the procedure “was so unnecessarily suggestive and conducive to irreparable mistaken identification that [the defendant] was denied due process of law.”

ii. Manson v. Brathwaite ; (1977) An undercover police officer identified a suspect from a photograph (only one photo offered for the ID) two days after the officer bought drugs from the suspect.

1. Rule: Even if a witness’s ID is based on suggestion and is unnecessary, it does not violate the D’s due process rights if it is reliable based on the totality of the circumstances.

a. ASK: Whether under the totality of the circumstances the identification was reliable even though the confrontation procedure was suggestive. (Ex: cop IDs D from a single photo array after buying drugs)

i. This Court adopts the rule that will rely on the totality of the circumstances and will permit the admission of the confrontation evidence, if, despite the suggestive aspect, the out-of-court identification possesses certain features of reliability.

ii. Factors to consider:1. Opportunity of the witness to view the criminal at the time of the crime;2. The witness’ degree of attention;3. The accuracy of his prior description of the criminal;4. The level of certainty demonstrated at the confrontation; and5. The time between the crime and the confrontation.

III. SUGGESTED RULES FOR MAXIMIZING ACCURACY OF EYEWITNESS IDENTIFICATIONS

1. Double-blind procedure: the person conducting the lineup or photographic display should not know who the suspect is.

2. Eyewitnesses should be told that it is possible the suspect may not be in the lineup or display so they do not feel compelled to make an identification.

3. The suspect should not be presented differently than the others in the lineup or display.4. When the eyewitness makes the identification, a statement should be taken regarding his degree of

confidence in the identification.

iv. The Duke Lacrosse Case; (2006) Six players alleged gang-raped exotic dancer. 13-months later, N.C. AG said all indicted players were innocent and all charges were dropped, AG disbarred for unethical conduct in using an “all suspects” photo review, where anyone chosen could be prosecuted b/c they were all lacrosse players who attended the party.

V. FINGERPRINTS 1. 24 Black men detained and fingerprinted based on rape Vs general description. S.C. ruled that, in

most cases, a party cannot be detained for fingerprinting or for participation in a lineup without probable cause to do so without probable cause would violate due process. Davis v. Mississippi

2. The Court has hinted that there might be some circumstances in which reasonable suspicion will be enough to warrant at least brief detention in the field for fingerprinting without violating due process. Hayes v. Florida

18. SCOPE OF THE EXCLUSIONARY RULES

A. STANDING TO OBJECT TO THE ADMISSION OF EVIDENCE I. HISTORICAL BACKGROUND

1. This concept usually refers to the legal right to object.

42

Page 43: Crim Pro Outline 1.1

2. Traditionally, to have standing to contest a search, a party must have had a reasonable expectation of privacy in the place to be searched. Thus, they could not object to evidence gained in illegal search of third party.

3. However, the concept of standing has checked in history,a. Before the Supreme Court conclusively ruled on the issue, some courts recognized the

concept that standing was required to challenge an illegal search.b. On the other hand, in some states such as California, the standing doctrine was

legislatively abolished.4. CASE LAW

a. United States v. Payner ; Gov’t stole a bankers briefcase and copied the information and attempted to admit it as evidence against 3rd party. The trial judge thought it was outrageous and did not allow the evidence in

i. S.C. Held: the ‘supervisory power does not authorize a federal court to exclude evidence that did not violate the defendant’s 4A rights [since it was the banker’s rights who were violated, not Payner’s]

1. Problem: takes admittedly illegal behavior and immunizes it.b. Simmons v. United States ; The testimony given by defendant in order to establish his

standing may NOT thereafter be used against him at trial on the issue of guilt. Were it otherwise, a defendant who wishes to establish standing must do so at the risk that the words which he utters may later be used to incriminate him.

5. EXPECTATION OF PRIVACY

a. Anyone with an expectation of privacy in the searched premises may challenge the search even though not present when the search was conducted, if cops try to use third party evidence against them b/c they were the fruits of an unauthorized search.

i. Olsen the Court held that overnight guests have sufficient expectation of privacy in the home to allow them to challenge the legality of a search of the home.

ii. The same is true for businesses; they are areas in which there is a reasonable expectation of freedom from gov’tal intrusion. Mancusi v. DeForte

II. THE MODERN APPROACH

1. Rakas v. Illinois ; Cops stopped and searched a car which Ps neither owned nor leased but were occupying as passengers. Cops violated none of THEIR rights. The inquiry is D-specific.

a. The Court held that standing should depend on whether the police action sought to be challenged is a search (i.e., a violation of legitimate expectation of privacy) with respect to the person challenging the intrusion.

b. The Court also stated that this ‘inquiry in turn requires a determination of whether the disputed search and seizure has infringed an interest of the individual D which the 4A was designed to protect.

2. Rawlings v. Kentucky ; The Court rejected the idea that a claim of ownership of property seized during a search of third party’s purse, confers upon the owner the automatic right to challenge the search. The Court specifically held that the D who prior to the arrival of the police had placed drugs in a companion’s purse, had no right to challenge a search of the purse because he had no legitimate expectation of privacy in the invaded place.

i. Mere ownership of the drugs found therein was not enough to confer “standing.”

3. Brendlin v. California (2007); Cops stopped car to check its registration w/o any grounds. S.C. held that when a cops makes a traffic stop he “seizes” a passenger as well as the driver “w/in the meaning of the 4th A” and thus a passenger, as well as the driver, may challenge the constitutionality of the stop.

4. Carter (1998) A police officer looked through a window and observed three people bagging cocaine, two of which were guests and now challenge the search as unreasonable.

a. Holding and Rulei. A person must be more than a casual day-guest in a home to challenge a

search of the home as being unreasonable under the 4A.ii. Per Rakas, in order for a D to claim the protection of the 4A, a D must

demonstrate that he personally has an expectation of privacy in the place searched, and that his expectation is reasonable. Property used for commercial purposes is treated differently for 4A purposes than residential property.

III. USE FROM ILLEGAL SEARCH AND SEIZURE VIOLATION

43

Page 44: Crim Pro Outline 1.1

1. United States v. Havens (1980) Two men smuggling cocaine by cutting holes in a t-shirt and using material to make pockets in other shirt to hold cocaine. Police catch one guy (with cocaine) and illegally search other guy (Havens) finding holey t-shirt in his luggage. When Havens was on witness stand, asked if he had anything to do with importing cocaine, Havens denied. Through questioning, prosecutor got holey t-shirt in by using it to impeach Havens testimony about lack of knowledge of smuggling. RULE: OK to use illegally gained evidence to impeach a lie in court.

IV. LIMITATIONS

1. DEFENSE WITNESS

a. Impeachment is not allowed of a defense witness with the use of illegally obtained evidence that he died hair to change appearance b/c evidence not used to prove witness was lying. James v. Illinois

2. PRIOR SILENCE

a. The Court has held that a prosecutor may not impeach a defendant with his POST-Miranda silence because Miranda contains the assurance that silence will carry no penalty. Doyle v. Ohio

b. HOWEVER, a prosecutor may use both post-arrest PRE-Miranda and pre-arrest silence.3. REBUTTING INSANITY

a. A defendant’s post-Miranda silence or request for an attorney CANNOT be offered at trial as substantive evidence to rebut his claim of insanity at the time of the commission of the crime. Wainwright v. Greenfield

B. “FRUIT OF THE POISONOUS TREE”I. GENERALLY: The ‘fruit of the poisonous tree’ doctrine is the idea that evidence obtained as a result of

unconstitutional government action (i.e., an illegal arrest, illegal search) cannot be admitted into evidence against the accused at trial. Evidence can tangible or verbal confessions.

ii. Wong Sun v. United States ; The Court recognized that ‘traditionally’ the exclusionary rule had barred only ‘physical, tangible materials,’ but concluded that ‘VERBAL evidence which derives so immediately from an unlawful entry and an unauthorized arrest is no less the “fruit” of official illegality than the more common tangible, PHYSICAL fruits of the unwarranted intrusion.’

1. The question is whether, granting establishment of the primary illegality, the evidence has been come at by exploitation of that illegality or instead by means sufficiently distinguishable to be purged of the primary taint.”

Ways to get around FoPT:- Inevitable discovery

III. APPLICATIONS

1. MIRANDA WARNINGS DO NOT PURGE THE TAINT OF AN ILLEGAL ARREST

a. Brown v. Illinois ; The exclusionary rule, when used to effectuate the 4A, serves interests and policies that are distinct from those it serves under the 5A Miranda warnings, and the exclusion of a confession made without them, do not alone sufficiently deter a 4A illegal arrest violation. The Brown-Dunaway Rule.

b. Taylor v. Alabama ; The Court applied Brown and held that a confession was the fruit of an illegal arrest and not admissible even though six hours had passed between the illegal arrest and the confession, the police advised Taylor of his rights three times, and Taylor was allowed to visit his friends before his confession.

2. PAYTON VIOLATION

a. Payton v. NY holds that the 4th A prohibits the cops from effecting a warrantless entry into a suspect’s home in order to make a routine felony arrest. Circumvention - OK to wait for guy to leave house to make felony arrest in public w/o warrant.

b. New York v. Harris ; the Court held that a statement made by a suspect outside his home is ADMISSIBLE even if the in-house arrest violated Payton if the police had probable cause to arrest the suspect. Harris’s statement taken at the station was not the product of being in unlawful custody, evidence was not ‘come at by exploitation of the D’s 4th A rights.’

i. Rationale of 4th A is to protect the home and anything incriminating the cops gathered from illegally arresting Harris in his home, which it did.

3. WARRANT SEARCH

44

Page 45: Crim Pro Outline 1.1

a. An illegal entry onto premises will not bar the use of evidence later found on the premises if police return with a warrant based wholly on information not obtained during the illegal entry and at that time discover the evidence. Segura v. United States

i. The same rule applies even when police observed the seized evidence during the illegal entry. Murray v. United States

4. TAINTED WITNESS

a. The testimony of a ‘tainted’ witness will be ADMISSIBLE, even if that testimony had been gathered via the illegal search of the envelope. United States v. Ceccolini

i. This is because it is difficult to prove that witness would not have voluntarily given the testimony to the police even without the search.

19. SPEEDY TRIAL AND OTHER SPEEDY DISPOSITION

A. SPEEDY TRIAL

i. Generally: States have their own statutory rules. For example, in Ohio, in a felony, a trial must be brought within 270 days of the indictment (if bail is not available for the defendant, then each day counts as 3 thus, 90 days).

ii. Barker v. Wingo ; (1972) D sought to set aside his murder conviction, claiming his right to a speedy trial was violated when the state, in order to first convict the defendant’s accomplice, sought and was granted, usually with no objection from the defense, 16 separate continuances over a five year period.

1. Holding and Rulea. In assessing whether a defendant had been deprived of his right to a speedy trial courts

must perform a balancing test that weighs the conduct of the D vs. the prosecution.b. The constitutional rules are a balancing test:

i. The rule is that the defendant’s assertion of or failure to assert his right to a speedy trial is one of the factors to be considered in an inquiry into the deprivation of the right.

ii. The entire responsibility on the prosecution must show that the claimed waiver of the speedy trial was knowingly and voluntarily made.

c. Factors to consider:i. Length of delay,

ii. Reason for delay,iii. The defendant’s assertion of his right, andiv. Prejudice to the defendant.

1. Prejudice should be assessed in how they affect the D’s speedy trial interests:

a. To prevent oppressive pretrial incarceration, b. To minimize anxiety and concern of the accused; and c. To limit the possibility that the defense will be impaired.

2. Notes and Considerations:a. A defendant who is in prison on one charge still has a right to a speedy trial of a second

charge.i. Defendant’s rights:

1. May want a concurrent sentence.2. The duration of his present imprisonment may be increased, and the

conditions under which he may serve his sentence greatly worsened, by the pendency of another criminal charge.

b. Vermont v. Brillon (2009); delay caused by the defense weighs against the D.

c. Consequences: Where there is a deprivation of the right to a speedy trial, dismissal of the indictment is the only possible remedy.

d. Interlocutory Appeali. The right to a speedy trial applies during an interlocutory appeal ONLY IF the

defendant is subject to indictment or restraint during that time.ii. A reasonable interlocutory appeal normally justifies delay.

45

Page 46: Crim Pro Outline 1.1

iii. Courts consider the strength of the position on appeal, the importance of the issue, and sometimes the seriousness of the crime.

e. When does the time begin to run? Under state statutes, time usually runs form the date the D 1st appears in court following his arrest.

B. THE RIGHT TO OTHER SPEEDY DISPOSITIONS

i. United States v. Lavasco (1977) The defendant moved to dismiss his federal indictment for possessing firearms stolen from the United States mails and for dealing in firearms without a license, because the indictment was filed more than 18 months after the offenses allegedly occurred.

1. Rule: A D is not deprived of due process simply because the criminal investigation delays his prosecution, even if his defense is prejudiced (Ex: indictment filed 18 months after offense occurred). Pre-indictment delay is completely irrelevant, speedy trial clause of 6th A only deals w/ after indictment or arrest.

a. As far as the Speedy Trial Clause is concerned a delay due to pre-indictment is wholly irrelevant, since the analysis of the Clause is that only “a formal indictment or information or else the actual restraints imposed by arrest and holding to answer a criminal charge engage the particular protections of the provision

2. Notes and Considerationsa. United States v. MacDonald ; The speedy trial clock stops running after the dismissal of

charges, and therefore does not run during the time after the dismissal of military charges and before indictment on civilian charges

i. The speedy trial clock stops at the dismissal of an indictment with prejudice, even if the government appeals the dismissal and the defendant continues to need counsel. United States v. Loud Hawk

b. If the defendant can show substantial prejudice resulting from a pre-indictment delay, the burden shifts to the government to show the reasonableness of the delay

i. If the government does not show reasonableness, the court must then balance the interests of the defendant and the public, considering the length of delay and the seriousness of the crime

c. The defendant need not prove intentional or reckless conduct by the government to show that a pre-indictment delay has violated his right to a speedy trial, but the court may consider such conduct when assessing whether there was a violation

20. GUILTY PLEASA. SOME ISSUES SURROUND PLEA BARGAINING

I. GENERALLY: In large urban counties, 90% of the cases are resolved via plea-bargaining. As a D, the only concern is what the outcome will be if the guilty plea is entered.

ii. Advantages to Plea Bargaining:1. Relieves court congestion,2. Relives prosecution and defense of inevitable risks of trial,3. Serves justice by tailoring sentences to individual cases.

iii. Two ways of looking at pleas: 1. Court will show leniency to people that admit guilt and forgo trial (reward), or2. If you insist on a trial you will pay rent for the courtroom in years (punishment).

iv. Booker ; In determining the sentence after trial or plea, the court can only look at prior record; can’t use some quantity or amount from the actual crime to determine the length of sentence.

v. Sentencing guidelines are used to impose some fairness into the system. They are not longer mandatory.

B. REJECTED, KEPT AND BROKEN BARGAINS; UNREALIZED EXPECTATIONS

i. Bordenkircher v. Hayes (1978); The prosecutor gave Hayes a choice between pleading guilty to forgery for a 5 year prison sentence, and risking trial and a recidivism indictment, which could result in a mandatory life sentence.

1. Rule: As long as a prosecutor has probable cause to support each charge, the choice of which charges to bring rests in his discretion, and his basing that choice on a desire to induce a guilty plea does not violate due process, it is a legislative tool. (OK to offer: Plea for 5 yrs. vs. trial get recidivism and possible 20 yrs.).

a. Problem is that this give the prosecutor too much leverage, authorized by law; might induce an innocent person to plead.

b. Prosecutor may not be vindictive when dealing in the plea bargaining process. In the give and take of plea-bargaining, there is no such element of punishment or retaliation so long as the accused is free to accept or reject the prosecutions offer.

46

Page 47: Crim Pro Outline 1.1

i. Prosecutor can’t change the indictment on appeal, but he can use his leverage as a tool.

ii. Can’t impose a greater sentence on a re-trial or on appeal unless there is intervening behavior.

2. WHAT CAN YOU PUT INTO A BARGAIN? OK TO GET CREATIVE!a. I will plead guilty if you don’t prosecute my wife – OK b. Give up your license to practice law in exchange we will let you plead to a misdemeanor

- OKc. We will dismiss the case in exchange for a charitable contribution - OKd. I will plead guilty to crimes I did not commit in order to get the correct sentence – OK e. Can a deal be to good? Yes and someone who is innocent may plead guilty.

3. NOTES AND APPLICATION

a. Brady Rule: A guilty plea must be both VOLUNTARY and INTELLIGENT to be valid.b. Intelligent Decision

i. However, an ‘intelligent’ decision to plead guilty does not require that the factors on which a defendant bases his decision be entirely error free.

1. Absent misrepresentation or other misconduct by the State, a guilty plea will stand even if the D misjudged the quality of the case against him or the penalties he might face.

ii. Information Needed:1. What the charge is the judge has to explain the elements of the

crime, particularly the mental state.2. What the sentence will be, or might be.

iii. HOWEVER, where the D has received misinformation about sentencing, his guilty plea will stand if he was aware of the actual sentencing possibilities.

iv. But, if a D enters a guilty plea because his counsel erroneously advised him that the prosecution had agreed to certain concessions as part of a plea bargain, that plea will be invalid because it would not be knowing and intelligent.

c. Voluntary decisioni. A choice is not involuntary because the options are unpleasant.

1. The fact that a prosecutor will give much for a guilty plea only shows how valuable the trial right is.

ii. The defendant must also be COMPETENT to plead thus, the defendant must have “a rational and factual understanding of the proceedings” and be “capable of assisting his counsel.” Godinez v. Moran

iii. A statute that authorizes a shorter sentence for a crime if a defendant forgoes his right to trial is constitutional.

iv. Rule: Neither an erroneous sentence estimate by D’s atty nor a D’s erroneous expectation, based on his atty’s erroneous estimate, renders a guilty plea involuntary.

II. JUDGES INVOLVEMENT

1. General Rule: Judges shall not participate in plea bargaining, Rule 11(c)(1). OK for the judge to indicate whether or not they would accept a sentence.

2. Sometimes it is the judge, rather than the prosecutor, who offers the defendant a plea bargain, which can raise some constitutional concerns.

a. People v. Dennis ; trial judge offered a definite sentence (2-6 yrs) in exchange for a guilty plea; when the post-trial sentence (40-80 yrs) was 20 times the sentence offered at the plea, the great disparity led to a reasonable inference of constitutional deprivation.

3. United States ex rel Elksnis v. Gilligan ; Some courts hold that the disparity between the positions of the judge and the accused are so great that no guilty plea bargain with a judge is voluntary.

a. A guilty plea predicated upon a judge’s promise of a definite sentence by its very nature does no qualify as free and voluntary. The plea is so interlaced with the promise that the one cannot be separated from the other.

47

Page 48: Crim Pro Outline 1.1

iii. Santobello v. New York (1971); According to a plea agreement, Santobello pleaded guilty in exchange for the prosecutor’s promise to make no sentencing recommendation, but a new prosecutor recommended the maximum sentence.

1. Rule: Plea-bargaining is essentially a contractual relationship, and the remedies will be as such, here, specific performance. S.C. says we will enforce the K, but the deal must be disclosed to the judge and on the record.

2. NOTES AND APPLICATION

A. IS THE PROSECUTOR FREE TO WITHDRAW HIS OFFER?i. Mabry v. Johnson ; As long as the D did not rely on a plea agreement to his

detriment, the prosecutor may withdraw it.

B. WHAT’S THE REMEDY FOR A D WHO WAS IMPROPERLY INDUCED TO REJECT A PLEA BARGAIN?

i. State v. Kraus ; If the D is induced to reject a plea, then convicted, the court will sometimes allow the D to plead to a lesser included offense.

C. D’S FAILURE TO PERFORM AS BARGAINED

i. Ricketts v. Adamson ; If a D fails to perform his part of a plea agreement on the assumption that his obligation is already complete, but aware that if his interpretation is wrong his failure would be a breach of the agreement, there is no double jeopardy violation if a court, finding the D in breach, returns the parties to the status quo and reinstates the original charges according to the terms of the agreement.

IV. UNREALIZED EXPECTATIONS

1. Must always make sure that any terms are on the record so they can be enforced . The judge can agree or let the D know that he may get more time, and then the D has a choice to make; take the plea or go to trial.

a. Remember, you cannot predict with certainty what your judge is going to do unless they tell you!

2. Fields v. Gibson ; D’s atty told him to enter a blind guilty plea w/ no promises or guarantees; D did so expecting a lesser sentence.

a. Rule: Neither an erroneous sentence estimate by D’s atty nor a D’s erroneous expectation, based on his atty’s erroneous estimate, renders a guilty plea involuntary.

b. NOTE that voluntariness does not depend on subjective expectations; the plea will be valid even if the subjective expectations are mistaken

3. Ex parte Griffin ; On the other hand, if a D enters a guilty plea because his counsel erroneously advised him that the prosecution had agreed to certain concessions as part of a plea bargain, that plea will be invalid because it would not be knowing and intelligent.

a. HOWEVER, if the defense counsel objectively f’s up, then the D may have a remedy under ineffective assistance of counsel.

C. PROFESSIONAL RESPONSIBILITY; THE ROLE OF THE PROSECUTOR AND DEFENSE COUNSEL

I. GENERALLY

1. Only the D can decide whether to plead guilty or not.2. Must asses the case and understand who you represent, the D and not the mom or dad.3. Only the defendant himself, and not his attorney, can waive the right to counsel.4. Where the defendant has not waived this right, the lack of effective counsel during the negotiation

of a plea agreement will make the plea constitutionally defective.

II. BELIEF OF GUILT OR INNOCENCE

1. United States v. Rogers ; Some courts hold that it is unreasonable for an atty to recommend that a D plead guilty although he believes himself innocent—regardless of the difficulties of proof and probability of conviction.

III. PROSECUTORIAL DISCRETION

1. Prosecutor must see that justice is done.2. Premo v. Moore ; (2011) D confessed to cops of accidently shooting V. W/ D’s atty’s advice, D

plead no contest to felony murder in exchange for the minimum 300 month sentence. D then claims ineffective assistance of counsel. Held: D’s prospects at trial were not certain, D’s atty made a reasonable choice to opt for a quick plea bargain.

48

Page 49: Crim Pro Outline 1.1

3. In criminal case you need to assess the immigration status of your client when advising on pleas. If the client enters a guilty plea on a deportable offense, what would the remedy be for the D? Today, anyone convicted of a felony who is in the country unlawfully is gone. If they are in the country lawfully, they may be gone depending on the felony.

4. Iowa v. Howe ; Prosecutor let people plead to lesser traffic violation and Iowa said this was not OK. OK to reduce speeding to bad equipment. Bluth said this should be OK b/c who is being hurt by this.

5. Newman v. United States (D.C. Cir. 1967); Newman and Anderson both committed the same offenses, but the prosecutor offered a plea bargain only to Anderson and not to Newman.

a. Rule: Even if two people have committed the same offense, a prosecutor has no duty to treat them alike as to charges. The prosecutor can and should take all factors into consideration.

6. United States v. Ruiz (2002); Ruiz was arrested in possession of drugs and was offered a “fast track” plea (D waives indictment, trial and appeal and in return, the government agrees to recommend a level-two downward departure from the applicable sentencing guidelines). Issue was whether the 5A and 6A require prosecutors, before entering into a binding plea agreement, to disclose “impeachment information relating to any informants or other witnesses.”

a. Rule: The Constitution does not require the prosecution to disclose all discovery information (impeachment info relating to informants & witnesses) before a plea bargain, only before a trail.

i. So, impeachment stuff may be withheld, but exculpatory stuff must be given. Need to get all of this stuff ahead of time to ensure the best trial, best decision.

7. In Re United States (7th Cir. 2003); Gov’t and D (racist cop) both want to dismiss the civil rights count w/ prejudice, but the district judge disagrees w/ justice departments exercise of prosecutorial discretion. Held: Judge must grant gov’t motion to dismiss the civil rights charge against racist cop. Court doesn’t get to play prosecutor.

IV. CHOICE OF PLEA

1. Federal System: Guilty, not Guilty and no Contest (w/ the courts consent) a. Procedure: D must be advised of his rights, the nature of each charge, maximum

penalty, mandatory minimum, special assessments. Before accepting the plea the court must accept the D personally and ask about any promises made.

b. Before entering judgment on the guilty plea the court must determine whether there is a factual basis for the plea. Is the judge going to let the D plea to something he says he did not do? (Mr. Johnson vs. Dr. Johnson rapist case, Mr. took the plea even though he said he was innocent).

i. North Carolina v. Alford ; D didn’t want to be sentenced to death, and even though he said he did not do it, he wanted to take the deal. Judge said that if he is saying he did not do it, he couldn’t plead guilty. In order for the judge to be satisfied that there is a factual basis for the plea the judge needs to listen to the prosecutor lay out the facts of the case. ALFORD PLEA

2. State system: No Contest (w/o the courts consent), guilty, not guilty

V. WITHDRAWAL OF PLEA 1. A defendant may move the court to withdraw his guilty plea before sentencing. If the court

has not yet decided whether to accept the plea agreement, it may deny the motion to withdraw the plea and accept the original plea regardless of the motion.

a. The standard is usually “any fair and just reason” for pre-sentence withdrawal motions and something higher, such as “manifest injustice,” for later motions.

VI. THE EFFECT OF A GUILTY PLEA 1. Generally, a counseled plea of guilty is an admission of factual guilt so reliable that, where

voluntary and intelligent, it quite validly removes the issue of factual guilt from the case. A guilty plea, therefore, simply renders irrelevant those constitutional violations not logically inconsistent w/ the valid establishment of guilt.

2. D pleading guilty must be informed of FRCP Rule 11 by the judge: the nature of each charge, any maximum possible penalty, including imprisonment, and term of supervised release, any mandatory minimum penalty, any applicable forfeiture, the court’s authority to order restitution, the court’s obligation to impose a special assessment, the court’s obligation to calculate the

49

Page 50: Crim Pro Outline 1.1

applicable sentencing guidelines range, the terms of any plea-agreement provision waiving the right to appeal.

3. What’s given up by the guilty plea? The right to plead not guilty, the right to a jury trial, the right to be represented by counsel, the right at trial to confront and cross-examine adverse witnesses, to be protected from compelled self-incrimination, to testify and present evidence.

a. If judge screws up on something substantial (like you will be deported) we need to fix it. Otherwise, substantial compliance is OK.

b. Federally, a guilty plea will waive any legal issues that could be brought on appeal. Ex: Take the plea but waive your right to appeal on 4th A issues.

c. At the State level, a D will always want to preserve the appellate issue by pleading no contest!!

4. After the plea is accepted, the case is referred for a pre-sentence investigation. If D changes his mind, he must show a fair and just reason for the w/drawal. After sentence is imposed, the D can’t w/draw plea, but would need to appeal.

5. McMann v. Richardson (1970) Where the D pleads guilty b/c of a prior “coerced” confession to save himself the expense and agony of trial (even though he thinks he might win) or to minimize the penalty, he can’t later challenge the plea.

6. Tollet v. Henderson (1973); D pleaded guilty to murder on advise of atty, sought to challenge b/c indicting grand jury was unconstitutionally selected.

a. Rule: A guilty plea, voluntarily and intelligently entered, may not be vacated b/c the D was not advised of every conceivable constitutional plea. (Ex: possible grand jury unconstitutionally selected).

7. Menna v. N.Y. (1975); OK to challenge a guilty plea where prosecution would have been barred by double jeopardy.

8. Bousley v. U.S. (1998); If D can show his plea was “unintelligent” b/c he, his counsel, and the court were misinformed as to the elements of the crime, he can challenge the plea as unconstitutional.

21. TRIAL BY JURY a. Right to Jury Trial

i. Right any time D is charged w/ serious offense. Some misdemeanors don’t carry the right.ii. Duncan v. Louisiana (1968); Black D convicted 2 years imprisonment via Bench Trial for slapping white

boys arm outside recently desegregated school. S.C. Incorporated 6th A right to trial by an impartial jury to the states via the 14th A.

b. Dimensions of the Right to Jury Trial i. Petty Offenses not subject to the 6th A jury trial right. Where max penalty is 6 months or less, or fine of

$5K or less, even if several petty offenses are joined, no right to jury. 1. Still have the right to counsel (any time they can put you in jail)

ii. Jury Size: 6-person jury minimum. Goals of jury are to promote group deliberation, free from outside attempts at intimidation, and to provide a fair possibility for obtaining a representative cross section of the community. Bluth says States prefer fewer b/c if it is 5-1, the 1 doesn’t have a chance.

iii. Jury Unanimity: 1. Apodaca v. Oregon (1972); In State cases it is OK for a substantial majority of the jury to be

convinced, ex: 11-1, 10-2, or 9-3. If there are only 6 jurors the verdict needs to be unanimous. a. General rule is that it has to be unanimous, only a few states allow for un-unanimous.b. Ohio says ¾ is enough for civil verdict.

iv. About what must a jury agree? 1. Schad v. Arizona (1991); OK to convict D w/o jury agreement as to which course or state of mind

actually occurred. Court will often ask the jury to decide the highest count first if there are multiple counts.

v. The Deadlocked Jury: 1. It is OK for judge to send jury back several times for deliberations. Ok for judge to dismiss jury

with “manifest necessity” after sufficient and reasonable time for deliberation when jury can’t reach verdict. “Dynamite charge” judge tells the jurors to get back there and listen to the majority!

vi. Jury Nullification: 1. You can’t tell the jury that they have an absolute right to ignore the law, but if they do too bad.

vii. Inconsistent Verdicts: 1. Verdict inconsistency itself is not constitutionally intolerable and a D may challenge on appeal the

sufficiency of evidence supporting any guilty verdict.

50

Page 51: Crim Pro Outline 1.1

viii. Right to Jury Sentencing? 1. No 6th A right to have the jury determine the sentence. Generally speaking, jurors tend to impose

harsher sentences than judges.

ix. Waiver of Right to Jury Trial: 1. The D does NOT have the right to insist on a bench trial. A D’s only constitutional right

concerning the method of trial is to an impartial trial by jury.

c. Jury Selection

i. Composition: Juries must be selected at random from a fair cross section of the communities in the district or division wherein the court convenes and no citizen shall be excluded from service as a juror on account of race, color, religion, sex, national origin, or economic status.

ii. Taylor v. Louisiana (1975); D challenges his conviction b/c women were excluded from the jury. 1. Rule: The fair cross-section jury requirement of the 6th A is violated by the systematic exclusion

of women. Although juries must be drawn from a source fairly representative of the community, the court imposes no requirement that juries actually chosen must mirror the community and reflect the various distinctive groups in the population. Ds are not entitled to a jury of any particular composition.

iii. Hamer v. U.S. (1958); D challenged conviction based on voir dire proceedings. S.C. found a court may permit the D to conduct the examination of prospective jurors or may itself conduct the examination. In the latter event the court shall permit the D to supplement the examination by such further inquiry as it deems proper or shall itself submit to the prospective jurors such additional questions by the parties or their attorneys as it deems proper. Only capital offenses require advance notice of jury lists.

1. Juror’s right to Privacy: The prospective juror’s right to privacy must yield to the imperatives of a fair trial, esp. in a murder case.

2. Who is able to ask the questions during voir dire? Depends on custom of the court. 3. Jury books? Not an issue in federal court b/c the jury population is too big. In smaller counties, it

might be more valuable. Always ask the jurors if they know the prosecuting attorney. 4. Jury questionnaire are commonly used. 5. Jury consultants do market research and come back w/ demographics of your perfect jury. Then

your job is to ask the perspective jurors these questions to try to get these types of pple on the jury. Very expensive so doesn’t happen all that often.

6. Questions: you can ask prospective jurors any question that is part of the issue and you can preempt people for BIAS, including race, sexual orientation.

iv. Challenges for Cause and Voir Dire Examination 1. The “For Cause” Standard = BIAS

a. Statutes often list the permissible grounds for a “for cause” juror challenge, “the juror has a state of mind that will prevent him from acting w/ impartiality.” Can’t be fair or impartial, they are BIAS.

2. Implied Bias: A court may decide the juror is biased b/c of;a. Employment: In a case against the gov’t, its EEs are not challengeable for cause solely

by reason of their employment.b. Membership in Organizations: Can’t challenge jurors for cause based upon

membership in NRA or NAACP.c. A Right to Ask Certain Questions: OK to ask about bias in general (racial bias), but

D’s rights were not violated when court refused to ask potential jurors how they felt about beards.

d. Voir Dire as Protection Against Jury Misconduct: i. The following constitute juror “misconduct” for which a conviction may be

overturned: 1. Discussing the case outside of jury deliberations, 2. Inspecting on their own the scene of the crime, 3. Conducting experiments, 4. Using a dictionary to define a term mentioned in the judge’s charge, 5. Lying during voir dire, and 6. Considering information about the case that comes from a source other

than the trial.

3. BATSON V. KENTUCKY (1986); D challenged his conviction b/c of prosecutors use of peremptory challenges to exclude blacks from the jury.

51

Page 52: Crim Pro Outline 1.1

a. Rule: Although a prosecutor ordinarily is entitled to exercise permitted peremptory challenges for any reason at all, as long as that reason is related to his view concerning the outcome of the case to be tried, D’s individual right to equal protection of the law is violated if the state intentionally excludes members of his racial group from the jury.

4. Peremptory Challenges a. Number of Challenges: 3 in Ohio, B/tw 5 and 10 in a felony case.b. Batson Rule: No citizen shall be excluded from service as a juror on account of race,

color, religion, sex, national origin, or economic status. Applies to both the defense and the prosecution.

c. Baston Three Step Analysis (race example)i. Procedure: Once the opponent of a peremptory challenge has made out a

prima facie case of racial discrimination (step 1), the burden shifts to the proponent of the strike to come forward with a race-neutral explanation (step 2). If a race-neutral explanation is tendered, the trial court must then decide (step 3) whether the opponent of the strike has proved purposeful racial discrimination.

1. Step One: Prima Facie Case; Must provide evidence sufficient to permit the trial judge to draw an inference that discrimination has occurred.

2. Step Two: Race-Neutral Explanation; Other side must present racially or gender neutral justification to the court.

3. Step Three: Assessing the Credibility of the Neutral Reason; Court must consider persuasiveness of neutral justification presented.

5. Can you get rid of a judge assignment? a. Look at how the judge knows, whether he has a financial interest. b. Monroeville , mayor got paid piecemeal for each issued, got shut down b/c of bias. Have

to file an affidavit of bias w/ S.C. justice of State – hard to prove.

22. THE CRIMINAL TRIAL

A. PRESENCE OF THE DEFENDANT – THE 6TH AMENDMENT

i. Generally: At trial the D has a right to be present and confront his accusers, the right to testify in his defense as well as the right to decline to testify. D has the right against improper statements to the jury by the prosecutor or judge.

ii. Illinois v. Allen (1970); A D who was removed from his trial for disrupting the courtroom appealed his conviction on the grounds that his 6A right to be present in the courtroom had been violated. Here, D waived his right to be present via his conduct.

1. Rules: A trial court may order a defendant removed from the courtroom, if necessary, in order to preserve order in the court. The flagrant disregard in the courtroom of elementary standards of proper conduct should not and cannot be tolerated.

a. The constitutionally permissible actions will be:i. Can put unruly D in isolation w/ TV and Bluetooth to atty

ii. Cite him for contempt (could be used strategically to delay trial)2. In Absentia:

a. Under the Federal Rules, a D may not be tried in absentia, that is a trial may not be commenced if the D is not present. Once the trial has started, if the D disappears, the trial can continue.

3. Pre-trial Proceedingsa. A defendant does not have the right to be present at all pretrial proceedings.b. Confrontation Clause: The CC requires that a D have the right to be present so the D

will have the opportunity to conduct a cross-examination of witnesses. If exclusion from a hearing does not interfere with the D’s right to cross-examine, it is permissible for a court to exclude the D.

c. Due Process: DP requires that a defendant be allowed to be present at any stage of the criminal proceeding if his presence would contribute to the fairness of the proceeding.

4. Appearance of Defendant:a. The circumstances of a defendant’s appearance may violate DP, e.g., when a defendant is

forced to appear before a jury in prison garb. If it is a jury trial and the D has street clothes he has to be allowed to wear them.

52

Page 53: Crim Pro Outline 1.1

iii. Deck v. Missouri (2005); At sentencing D wore leg irons, handcuffs, and a belly chain which were visible to the jury and is challenging his death sentence. S.C. held that courts could not routinely place Ds in shackles or other physical restraints visible to the jury during the penalty phase, as it forbids there use during the guilt phase, of a capital proceeding. However, the constitutional requirement was not absolute. It permitted a judge, in the exercise of her discretion, to take account of special circumstances, including security concerns, that may call for shackling; court must put concerns on the record.

iv. Michigan v. Bryant (2011); At D’s trial the court admitted statements V made to the cops who discovered him mortally wounded in a gas station parking lot. D said Confrontation Clause barred the admission of V’s statements to the cops. S.C. held that the circumstances of the interaction b/tw V & cops indicate that the primary purpose of the interrogation was to enable police assistance to meet an ongoing emergency. Therefore, V’s ID and description of the shooter and the location of the shooting were not testimonial statements and their admission at trial did not violate the CC b/c was description of current events surrounding the emergency.

1. Not a dying declaration b/c the officer did not say “do you understand you’re dying, who did it.”

B. RIGHT TO REMAIN SILENT

i. Griffin v. California (1965) D appealed his murder conviction claiming that the prosecution and the court’s comments on his refusal to testify violated his 5A right against self-incrimination.

1. Rule: The 5A’s protection against self-incrimination prohibits the prosecution and the court from commenting upon a defendant’s decision not to testify at trial in a criminal case.

2. Notes and Applicationa. Lockett v. Ohio ; When the defense promised the jury that the D would testify, if the D

does not later testify it may be permissible for the prosecution to note that its case was “un-refuted” and “uncontradicted.”

b. United States v. Robinson ; A prosecutor may respond to a defense argument that the D did not have the opportunity to share his side of the story by noting to the jury that a D has the right to testify.

II. ORDER OF TESTIMONY

1. Brooks v. Tennessee ; The Constitution allows for the defendant to decide when he wants to testify, if at all. The defendant ought not to have to make this choice until all of the testimony is in.

C. COUNSEL’S ARGUMENTS

I. IMPROPER ARGUMENT

1. Darden v. Wainwright (1986) A prisoner sought federal habeas corpus claiming that a prosecutor’s improper closing comments had violated his DP right to a fair trial. Here, the defense opened up the door inviting the remarks and the defense did not object during the prosecutor’s closing.

a. Rule: A prosecutor’s improper comments will not violate a D’s right to a fair trial UNLESS it is found that the comments rendered the defendant’s trial fundamentally unfair. The closing argument is meant to comment on the evidence which has been presented personal belief about how the outcome of the case will be is NOT allowed.

2. Types of improper arguments:a. Statements of opinion as to the credibility of witnesses or the guilt of the accused (must

talk about the evidence b/c this is what proves guilt).b. Arguments intended to divert the jury from the issues and evidence presented in a trial by

injecting issues beyond the guilt or innocence of the accused,c. Arguments meant to inflame the passions or prejudices of the jury,d. Arguments that question the integrity of defense counsel,e. A reference to the possibility a defendant may receive a lenient sentence.

II. STANDARD OF REVIEW – POSSIBLE PREJUDICE

1. INVITED RESPONSE

a. United States v. Young ; While a prosecutor should not engage in improper argument even in response to improper defense argument, on appellate review, a court will be less likely to overturn a conviction if it finds that a prosecutor’s argument was meant to “right the scale.”

2. NECESSITY OF OBJECTION

a. Unless an improper argument is particularly egregious, it is necessary for an attorney to object in order to preserve an issue for appeal. MUST BE ON THE RECORD!

III. SANCTIONS FOR IMPROPER ARGUMENT

1. REVERSAL

53

Page 54: Crim Pro Outline 1.1

a. United States v. Modica ; Unless an argument results in a violation of DP, reversal of conviction is an inappropriate sanction.

2. Trial Court’s Rolea. A trial court should exercise its authority to ensure that attorneys do not engage in

improper argument by:i. Striking the argument and instructing the jury,

ii. Reprimand the attorney, oriii. Refer the attorney to a grievance committee.

23. DOUBLE JEOPARDY

a. Generally: Under the 5A, a criminal defendant is guaranteed that he is immune from criminal prosecution more than once for essentially the same crime. ‘Nor shall any person be subject to the same offense or shall twice be put in jeopardy for life and limb.’

I. QUESTIONS

1. Under what circumstances can a D be tried a 2d time?2. Under what circumstances can a D NOT be tried a 2d time?

II. PURPOSES

1. Prevent the government from continuing to try a D when a not guilty verdicts result.2. Let the D know that he will only have to “run the gauntlet” once.

III. WHEN JEOPARDY ATTACHES 1. In jury trials, jeopardy attaches when the jury is “impaneled and sworn.”2. In bench trials, it attaches when the first witness is sworn.

B. RE-PROSECUTION AFTER A MISTRIAL i. Illinois v. Somerville (1973); A mistrial was granted over D’s objection based on a defective indictment

(insufficient to charge a crime, would be overturned on appeal). D claimed double jeopardy, but was subsequently re-tried under a valid indictment and convicted. In a civil case you can amend the complaint, in a criminal case the grand jury decides what is in the complaint and they did not find one of the elements b/c it was not in the indictment.

1. Holding and Rules:a. RULE: A declaration of mistrial over a D’s objection does not prevent a second trial

where there is a “manifest necessity” for the mistrial or it is otherwise required by the “ends of public justice.” If it is manifestly necessary to end a trial, then jeopardy does not attach.

b. A trial court properly exercises its discretion to declare a mistrial if an impartial verdict cannot be reached, or if a verdict of conviction could be reached but would have to be reversed on appeal due to an obvious procedural error in the trial.

i. If an error would make reversal on appeal a certainty, it would not serve the ends of public justice to require that the Government proceed with its proof, when, if it succeeded before the jury, it would automatically be stripped of that success by an appellate court.

ii. Renico ; (2011) Criminal trial, jury sends note to judge, ‘judge, what if we can’t agree.’ Judge wants to know if there is a unanimous verdict, jury says no, and judge declares a mistrial. Judge was to quick to declare a miss-trial, and should have asked the jury to go back and listen to each other, given them more time, could have asked the D if he wants to start all over or if he wants the jury to go back and work some more. Historically there were two elements of double jeopardy: 1) sparing the D the burden of running the gauntlet twice. HELD: S.C. said judge was wrong and he should not have pulled the trigger and ended a trial, but jeopardy does not bar a second trial. Manifest necessity is basically anything the trial judge thinks is necessary to move the trial along. An appeal is a waiver of a miss-trial.

iii. U.S. v. Perez ; Once jeopardy attaches, you cannot grant a mistrial over the objection of the defendant, unless the mistrial is due to a hung jury. In the absence of a verdict (hung jury), jeopardy is said to be “continuing.” Under this continuing jeopardy principle, retrial after a jury has failed to reach a verdict is not a new trial but part of the same proceeding.

iv. Thompson v. United States mistrial after juror disqualified OK.v. Lavato v. New Mexico mistrial after defendant forgot to plead OK.

vi. Downum v. United States cannot re-prosecute after dismissal because witness failed to appear.vii. United States v. Jorn judge’s decision to grant mistrial to allow tax-fraud witnesses to confer with atty

regarding their liability for testifying was not “necessity,” because judge could instead have delayed trial.1. This prevents prosecutors from forcing mistrials to avoid a final acquittal.

VIII. ALTERNATIVES TO MISTRIAL

54

Page 55: Crim Pro Outline 1.1

1. Arizona v. Washington ; S.C. rejected the contention that the trial judge must make explicit findings as to the need for a mistrial in light of alternatives, but did insist that there be apparent from the trial record a “sufficient justification” for the mistrial ruling that reflected consideration of those alternatives.

a. The strictest scrutiny is appropriate when the basis for the mistrial is the unavailability of critical prosecution evidence or when there is reason to believe the prosecutor is using the superior resources of the State to harass or to achieve a tactical advantage.

2. The need to maintain the “appearance of impartiality” justifies granting a mistrial, even if the error benefits the defendant.

IX. DEFENDANT’S REQUEST FOR MISTRIAL 1. Generally, if the D asks for a mistrial or an appeal, his double jeopardy rights are waived.2. Oregon v. Kennedy (1982); The D’s motion to dismiss due to double jeopardy was granted after

the prosecutor provoked the D into moving for a mistrial during his first trial.a. Exception to the General Rule: The circumstances under which a D who has

successfully moved for a mistrial may invoke the bar of double jeopardy in a second effort to try him are limited to those cases in which the conduct giving rise to the successful motion for a mistrial was INTENDED to PROVOKE the D into moving for mistrial.

C. RE-PROSECUTION AFTER ACQUITTAL i. United States v. Scott (1978); An indictment against a drug-dealing cop was dismissed at D’s urging on the

grounds of prejudice, and the government was prohibited from appealing the dismissal based on double jeopardy.

1. Rule: Where the DEFENDANT himself seeks to have the trial terminated without any submission to either judge or jury as to his guilt or innocence, an appeal by the government from his successful effort to do so is NOT barred by double jeopardy.

2. Rule: If the judge dismisses based on a legal issue (law doesn’t apply or unconstitutional), not on the facts of the case, DJ does not bar re-trial.

3. Rule: A D is acquitted only when the ruling of the judge, whatever its label, actually represents a resolution in the defendant’s favor, correct or not, of some or all of the factual elements of the offense charged.

a. This will be the only instance in which double jeopardy will attach .

II. ERRONEOUS RULINGS 1. Rule: When a D has been acquitted at trial, he may not be retried on the same offense, even if the

legal rulings underlying the acquittal were erroneous.a. Sanabria v. United States ; bookie walks free after judge mistakenly decides horse racing

is not “game of competition.”

III. JUDGMENT N.O.V. 1. Rule: If the jury finds the defendant guilty, but later the judge orders an acquittal notwithstanding

the verdict, retrial is PERMISSIBLE.a. United States v. Wilson ; after jury convicts, judge grants earlier motion to dismiss for

pre-indictment delay. OK for gov’t to appeal order favoring D since the D wont’ be exposed to multiple trials.

D. RE-PROSECUTION FOLLOWING A CONVICTION

i. Lockhart v. Nelson (1988); A reviewing court set aside the D’s conviction because certain evidence was erroneously admitted against him, and further held that the Double Jeopardy Clause forbade the state to retry him. S.C. said no double jeopardy bar.

1. Rule: In a case where the evidence offered by the State and admitted by the trial court—erroneously or not—would have been sufficient to sustain a guilty verdict, double jeopardy does not preclude retrial.

a. Burks v. United States When a D’s conviction is reversed on appeal on the sole ground that the evidence was insufficient to sustain the jury’s verdict, then retrial on the same charge is barred. OK to retry on other (or lesser) charges. If the evidence is totally wacky (no evidence or they missed an element totally) double jeopardy kicks in, if the evidence is only weak, double jeopardy won’t attach.

b. D charged w/ aggravated murder, jury finds her guilty of murder. Murder conviction overruled. What can she be re-tried on? S.C. said if they come back with murder when they are charged w/ aggravated murder = implied acquittal. D can’t be re-tried on the aggravated count, but can be re-tried on murder.

55

Page 56: Crim Pro Outline 1.1

II. SENTENCING 1. Sentencing decisions are not covered by double jeopardy

a. North Carolina v. Pearce ; If the D’s conviction is reversed, but he is later retried and re-convicted, and ends up sentenced to a heavier term than he got for the original conviction, the higher sentence is NOT BARRED by double jeopardy. But, after a D succeeds in winning retrial or resentencing on appeal, a judge cannot impose a higher sentence to retaliate against the D for the successful appeal of the first judgment.

i. This was a due process violation, not a double jeopardy issue.ii. Guy gets sentenced to 10 years, appeals the sentence, gets retried, then gets 20

years not allowed UNLESS the court has new objective reasons for higher sentence (mis-behaving in prison).

b. Sentencing is not part of double jeopardy, and it is totally reviewable on both sides.

E. RE-PROSECUTION BY A DIFFERENT JURISDICTION

i. Heath v. Alabama (1985); The D was tried and convicted in an Alabama for murdering his wife after he had already been convicted and sentenced in Georgia for the same crime.

1. Rule: The dual sovereignty doctrine compels the conclusion that successive prosecutions by two States for the same conduct are NOT BARRED by double jeopardy.

2. Rationale: When a D in a single act violates the “peace and dignity” of two sovereigns by breaking the laws of each, he has committed two distinct “offenses.”

a. Same rule applies in terms of federal and state sovereignty,i. If there is adequate prosecution at the state level, the federal gov’t will

not prosecute. It all depends on whether or not the V is satisfied.ii. However, this will not happen between a city and a state.

b. From mayor’s court to municipal court, double jeopardy will not apply

24. SENTENCINGA. PURPOSE OF PUNISHMENT

i. Generally, for most Ds, sentencing is what the case is really about. B/c of its pervasive influence it is the most important aspect of the criminal process to understand.

ii. Punishment’s major objectives: Rehabilitation, Deterrence, Incapacitation, and Retribution.iii. A Swing Back Towards rehabilitation?

1. Moving away from the philosophy that the punishment should fit the crime toward one that the punishment should fit the criminal.

2. Budget concerns have forced many states to re-think corrections policies. iv. All sentences are subject to review and we litigate sentences aggressively. We argue whether restitution is

appropriate, whether they pre-sentence report got the facts rights, etc. Judges decision is reviewable by both sides.

B. TYPES OF SENTENCES

I. CAPITAL PUNISHMENT 1. Two step process: Jury determines guilt or innocence, and then they determine whether or not to

impose death.2. Most severe punishment is execution. 3. Jurors are instructed that they must agree that the aggravating factors have been established

beyond a reasonable doubt before they can impose a death sentence.II. INCARCERATION

1. Most people we lock up, and we lock up the most per capita of any other country.2. Indeterminate Sentence: Includes a max and min term set by the judge w/in legislated limits, and

leaves to the parole board the task of determining precisely when the D will be released. Discretionary parole is in decline.

3. Determinate Sentence: Mandatory release after a fixed term set by the judge; early release through discretionary parole is not available.

4. Fairness says that similar D’s with similar crimes and backgrounds should be sentenced equally.III. PROBATION OR COMMUNITY RELEASE

1. Probation is release from custody with conditions. Conditions must be reasonably related to the offense, the rehabilitation of the D, or the protection of the public. If D violates the terms of release, he may then be sentenced to incarceration. Constitutional protections of a trial do not apply to probation revocation hearings.

IV. INTERMEDIATE PENALTIES, “PROBLEM SOLVING COURTS,” AND FINANCIAL SANCTIONS 1. Popular due to prison overcrowding, include: boot camp, house arrest, home confinement w/

monitoring devices, day reporting programs, fines, restitution and forfeiture.

56

Page 57: Crim Pro Outline 1.1

C. ALLOCATING AND CONTROLLING SENTENCING DISCRETION i. Gross disparities in the sentences for similar offenses, particularly differences linked to the race of the

offender or the ID of the judge, prompted a nationwide end to discretionary sentencing.

1. MANDATORY MINIMUM SENTENCES a. Statute requires a judge to sentence the D to a term at least as long as the term specified

in the statute. b. Generally deny the judge the legal power to depart downward, no matter how unusual the

special circumstances that call for leniency. Transfer sentencing power to the prosecutors, who can determine the sentences through the charges (not the intent of Congress).

2. SENTENCING GUIDELINES a. Created by sentencing commissions authorized by the legislature. The sentence in a

guidelines system is determined through the use of a sentencing grid that sets the presumed sentence range for each particular case.

b. One axis of the grid is a ranking of the criminal history of the offender, based on past convictions. The other axis ranks the severity of the particular offense.

c. Guidelines can be voluntary (not binding). In mandatory guideline systems the court may depart from the range recommended only if it justifies that departure w/ permissible reason.

3. APPELLATE REVIEW a. In jurisdictions w/ guideline or presumptive sentencing, the factual and legal conclusions

of the trial court underlying each sentence are part of the record and can be tested on appeal by either party for compliance w/ statutory command or guideline requirements.

D. CONSTITUTIONAL LIMITS ON SENTENCING PROCEDURE I. INFORMATION CONSIDERED IN SETTING THE SENTENCE – DUE PROCESS AS APPLIED TO SENTENCING

1. Williams v. New York (1949); D found guilty of murder in connection w/ robberies. Jury recommended life in prison. The judge, considering information obtained outside the courtroom (presentencing report), sentenced D to death. D is challenging the death sentence as a violation of his Due Process rights in that no person shall be tried and convicted of an offense unless he is given reasonable notice of the charges against him and is afforded an opportunity to examine adverse witnesses. (See Gardner below).

a. Rule: Judges use of probation department reports in sentencing does not violate the DPC, but aid judges in efforts to improve the administration of criminal justice. But judge can’t use uncharged criminal conduct b/c D can’t defend against it unless there is a trial. DPC means that the State must prove uncharged crimes, considered in sentencing, beyond a reasonable doubt.

ii. Defendant’s Exercise of Procedural Rights 1. D does not have the right to commit perjury and it is OK for judge to enhance sentence due to

perjury.2. Mitchell v. U.S. (1999); D plead guilty and did not speak at sentencing, and judge increase her

sentence inferring her silence adversely. a. Rule: It is a violation of a D’s 5th A privilege against self-incrimination for a judge to

draw an adverse inference from D’s silence at sentencing. iii. Constitution does not say that proof must be beyond a reasonable doubt, but it comes in through the DPC.

E. NOTICE, CONFRONTATION, DEFENSE SUBMISSIONS i. Gardner v. Florida ; (1977) The Constitution forbids a judge from sentencing a D to death w/o disclosing

those portions of the presentence report that form the basis for the sentence.

F. THE ASSISTANCE OF COUNSEL i. The 6th A right to counsel extends to sentencing, a stage of a criminal proceeding where substantial rights

of a criminal accused may be affected.

G. JURY TRIAL AND BURDEN OF PROOF

i. McMillan v. Pennsylvania (1986); S.C. rejected a challenge to a PA statute that mandated a 5-yr min sentence for any D convicted of one of several specified serious felonies, if a judge found at sentencing that the D visibly possessed a gun.

1. RULE: It is OK for the court to use facts not proven beyond a reasonable doubt or presented to a jury in sentencing (sentencing factor), even though it is not an element of the crime.

II. THE APPRENDI REVOLUTION

57

Page 58: Crim Pro Outline 1.1

1. Apprendi v. New Jersey (2000); D agreed to plead guilty to possession of gun for unlawful purpose, an offense punishable by 5-10 years. Cop said it was hate crime (white guy fires into black house) and judge found intent to intimidate and sentenced D to 12 years under NJ hate crime statute. D challenged sentence.

a. RULE: Other than the facts of a prior conviction, any fact that increases the penalty for a crime beyond the prescribed statutory maximum must be submitted to a jury, and proved beyond a reasonable doubt.

2. APPRENDI AND MANDATORY MINIMUM SENTENCING STATUTES a. Harris v. United States (2002); Apprendi does not apply to facts triggering mandatory

minimum sentences. The jury’s verdict authorizes the judge to impose any sentence below the minimum, considering any facts to do so. That a fact affects the D’s sentence, even dramatically so, does not by itself make it an element.

i. RULE: If a fact increases the mandatory minimum, but does not extend the sentence beyond the statutory maximum, it does not have to be presented to a jury or proved beyond a reasonable doubt.

3. APPRENDI AND DEATH SENTENCING a. Ring v. Arizona (2002); Under AZ law no death sentence could follow a jury’s decision

to convict a first-degree murderer without the further finding of an aggravated circumstance. Apprendi requires that the jury determine this aggravating circumstance.

4. APPRENDI AND SENTENCING GUIDELINES a. Blakely v. Washington (2004); Statutory max 53 months, judge sentence to 90 months

b/c he felt D acted w/ “deliberate cruelty,” a statutorily enumerated ground for departure in domestic violence cases.

i. RULE: The statutory maximum for Apprendi purposes is the maximum sentence a judge may impose solely on the basis of the facts reflected in the jury verdict or admitted by the D. When a judge inflicts punishment that the jury’s verdict alone does not allow, the jury has not found all the facts which the law makes essential to the punishment, and the judge exceeds his proper authority.

iii. United States v. Booker (2005); D convicted of possessing 50 grams of crack, sentenced based upon judge knowing he possessed 566 crack, D got 10 years longer than max of jury conviction. Here, judge uses uncharged behavior to increase sentence. S.C. reaffirms Apprendi: Any fact (other than a prior conviction) which is necessary to support a sentence exceeding the max authorized by the facts established by a plea of guilty or a jury verdict must be admitted by the D or proved to a jury beyond a reasonable doubt.

1. Application of Booker (plead to 50 grams, sentenced for 566 grams): If you plea to a lesser charge or amount you are covered under Booker to a sentence using the lower amount, they can’t sentence you for a higher amount.

25 multiple choice, each worth 2 points 10 short essays, give answer and reason w/ rule, one side of one blue book page w/o skipping lines should be enough to answer

each question, each worth 5 points

58